9310855720

Enroll Now
Student Login
Enroll Now
Student Login

9310855720

Prelims Solved Paper(2022)
Home >   Prelims

2022

Solved Questions with Answer

Q1. ‘’Rapid Financing Instrument’’ and ‘’Rapid Credit Facility’’ are related to the provisions of lending by which one of the following?

 a) Asian Development Bank

 b) International Monetary Fund

c) United Nations Environment Programme Finance Initiative

 d) World Bank

Show Answer

Answer: b

 Explanation:

The International Monetary Fund (IMF) offers quick financial support through its Rapid Financing Instrument (RFI), a lending tool. All member nations with an immediate need for balance of payments can access it. The RFI was developed in conjunction with a larger overhaul to increase the flexibility of the IMF's financial assistance to better meet the various demands of its member nations. The IMF's prior emergency assistance program was superseded by the RFI, which is applicable in a variety of situations.

• Where a full-fledged economic program is neither essential nor possible, low-income countries (LICs) experiencing an urgent balance of payments (BoP) requirement can get fast, concessional financial assistance from the IMF's Rapid Credit Facility (RCF) with no ex-post conditions.

Therefore, option (b) is the correct answer.

Q2. With reference to the Indian economy, consider the following statements:

 1. An increase in Nominal Effective Exchange Rate (NEER) indicates the appreciation of rupee.

 2. An increase in the Real Effective Exchange Rate (REER) indicates an improvement in trade competitiveness.

 3. An increasing trend in domestic inflation relative to inflation in other countries is likely to cause an increasing divergence between NEER and REER.

Which of the above statements are correct?

a) 1 and 2 only

 b) 2 and 3 only

c) 1 and 3 only

 d) 1, 2 and 3

Show Answer

Answer: c

 Explanation:

• The unadjusted weighted average rate at which the currency of one nation is exchanged for a basket of many foreign currencies is known as the nominal effective exchange rate, or NEER. The amount of local currency required to buy foreign currency is known as the nominal exchange rate. Should a NEER be considered to appreciate when it rises in value relative to a basket of other currencies under a floating exchange rate regime. The NEER depreciates if the home currency declines in value relative to the basket. Therefore, statement 1 is true.

• The weighted average of a nation's currency for an index or basket of other major currencies is known as the Real Effective Exchange Rate, or REER. By comparing the relative trade balance of a nation's currency with that of every other nation in the index. An increase in a country's REER indicates that imports are getting cheaper and exports are getting more expensive. As a result, it becomes less competitive in commerce. Therefore, assertion 2 is untrue.

• REER continues to track inflationary trends; recent observations in India indicate that REER is biased upward as a result of inflation. An increasing gap between NEER and REER is expected to result from an increase in domestic inflation relative to inflation in other nations. The growing current disparity in NEER and REER developments can be attributed to India's greater internal inflation rate in comparison to the six major currencies under consideration. Thus, statement 3 is true.

Therefore, option (c) is the correct answer.

Q3. With reference to the Indian economy, consider the following statements:

1. If the inflation is too high, Reserve Bank of India (RBI) is likely to buy government securities.

 2. If the rupee is rapidly depreciating, RBI is likely to sell dollars in the market.

 3. If interest rates in the USA or European Union were to fall, that is likely to induce RBI to buy dollars. Which of the statements given above are correct?

 a) 1 and 2 only

 b) 2 and 3 only

c) 1 and 3 only

d) 1, 2 and 3

Show Answer

Explanation:

• To streamline the money supply and interest rates, the Reserve Bank of India (RBI) purchases and sells government securities on behalf of the Center in a process known as open market operations, or OMOs. The objective of OMOs is to regulate the money supply or current liquidity in the economy. If In the event of inflation, the RBI implements a contractionary monetary policy, which involves selling government assets and removing surplus funds from the banking system. During a recessionary trend, the RBI is eager to increase the amount of money in circulation and guarantee sufficient credit availability for investment and output. To increase the money supply, it purchases securities. Thus, the first statement is untrue.

• Changes in interest rates in a foreign economy may prompt the RBI to take action. Interest rates will rise in the US if the Federal Reserve shrinks the amount of money in circulation. Financial Institutional Investments (FII) into India will decline as a result, as US debt investments will have grown increasingly alluring. The rupee would weaken as a result of decreased demand from international institutional investors who are showing less interest in holding it. In reaction, the RBI may decide to support the Indian rupee by selling dollars from its reserves. However, foreign institutional investors (FIIs) will increase their investments in India if interest rates in the US or the EU decline. The demand for rupees that arises will lead to an appreciation of the rupee. The RBI will buy dollars and add rupees to the system in retaliation. As a result, statements 2 and 3 are true.

Therefore, option (b) is the correct answer.

Q4. With reference to the “G20 Common Framework”, consider the following statements:

1. It is an initiative endorsed by the G20 together with the Paris Club.

 2. It is an initiative to support low-income countries with unsustainable debt.

Which of the statements given above is/are correct?

 a) 1 only

 b) 2 only

 c) Both 1 and 2

d) Neither 1 nor 2

Show Answer

Answer: c

Explanation:

• The G20 and the Paris Club have supported the Common Framework for Debt Treatment beyond the Debt Service Suspension Initiative (DSSI) as a structural means of assisting Low-Income Countries facing unmanageable debt. The unified structure brings the Paris Club and G20 official bilateral creditors together in a coordinated approach, marking a significant advancement for official creditors. Therefore, both claims 1 and 2 are true.

• Three nations have so far asked for a Common Framework treatment, and the first Creditor Committee for debt treatment for Chad was established in April 2021.

Therefore, option (c) is the correct answer.

Q5. With reference to the Indian economy, what are the advantages of “Inflation-Indexed Bonds (IIBs)”?

 1. The government can reduce the coupon rates on its borrowing by way of IIBs.

2. IIBs provide protection to the investors from uncertainty regarding inflation.

 3. The interest received as well as capital gains on IIBs are not taxable.

Which of the statements given above are correct?

 a) 1 and 2 only

 b) 2 and 3 only

c) 1 and 3 only

 d) 1, 2 and 3

Show Answer

Answer: a

 Explanation:

• Securities known as inflation-indexed bonds, or IIBs, are made to help shield investors against inflation. Because IIBs are inflation-indexed, principal and interest payments fluctuate in tandem with the rate of inflation. By using IIBs, the government can lower the coupon rates on its borrowing. Thus, the first assertion is true.

• Over the bond's life, an inflation-indexed bond shields issuers and investors against the risk of inflation. Therefore, statement 2 is true.

• Current tax laws will apply to capital gains and interest payments on IIBs. These bonds won't receive any preferential tax treatment. Therefore, assertion 3 is untrue.

Therefore, option (a) is the correct answer.

Q6)With reference to foreign-owned e-commerce firms operating in India, which of the following statements is/are correct?

 1. They can sell their own goods in addition to offering their platforms as market-places.

 2. The degree to which they can own big sellers on their platforms is limited.

 Select the correct answer using the code given below:

 a) 1 only

b) 2 only

 c) Both 1 and 2

 d) Neither 1 nor 2

Show Answer

Answer: b

 Explanation:

• The FDI Policy forbids enterprises with FDI from engaging in multi-brand retail trading activities, including retail trading through e-commerce. Multi-brand retail commerce refers to the sale of distinct goods under multiple brands on a single platform. FDI is not permitted in India. in e-commerce models that rely on inventories. The inventory model is used by Amazon and Walmart in the US, where an online retailer owns the products and services and sells them directly to retail customers. The primary goal of the restriction is to safeguard India's enormous unorganized retail market, which lacks the power to make massive purchases and provide substantial discounts. Therefore, assertion s

• In 2018, India modified its Foreign Direct Investment (FDI) regulations pertaining to e-commerce marketplaces, designating as "controlled" any vendor that contributes more than 25% of the platform's overall sales revenue. Therefore, no seller on any international e-commerce site may take more than 25% of the whole business. Therefore, statement 2 is true.

Therefore, option (b) is the correct answer.

Q7. Which of the following activities constitute real sector in the economy?

 1. Farmers harvesting their crops

 2. Textile mills converting raw cotton into fabrics

 3. A commercial bank lending money to a trading company

 4. A corporate body issuing Rupee Denominated Bonds overseas

 Select the correct answer using the code given below:

a) 1 and 2 only

 b) 2, 3 and 4 only

 c) 1, 3 and 4 only

d) 1, 2, 3 and 4

Show Answer

Answer: a

 Explanation:

• The real sector of an economy is the most important part since its operations influence economic production and are represented by the economic segments that are vital to the expansion of the GDP of the nation. Thus, in this sense, textile mills and farmers harvesting crops

• The real sector of an economy includes the process of turning raw cotton into fabrics. It is made up of people, nonprofit organizations that support households, and enterprises (nonfinancial corporations). From the perspective of economic and financial data, families and charitable organizations that support them are occasionally merged into a single subsector called "Other resident sectors."

• The real sector does not include a commercial bank providing funding to a trading firm or a corporate entity issuing bonds abroad denominated in rupees.

Therefore, option (a) is the correct answer.

Q8. Which one of the following situations best reflects “Indirect Transfers” often talked about in media recently with reference to India?

a) An Indian company investing in a foreign enterprise and paying taxes to the foreign country on the profits arising out of its investment

 b) A foreign company investing in India and paying taxes to the country of its base on the profits arising out of its investment

 c) An Indian company purchases tangible assets in a foreign country and sells such assets after their value increases and transfers the proceeds to India

 d) A foreign company transfers shares and such shares derive their substantial value from assets located in India

Show Answer

Answer: d

 Explanation:

• When foreign entities own shares or assets in India, the shares of those foreign entities are transferred rather than the underlying assets in India directly. This is referred to as an indirect transfer.

• A transfer of shares of a foreign firm or an interest in any entity established or registered outside of India is usually referred to as a "Indirect Transfer" if the shares or interest directly or indirectly derives its significant value from assets located in India. If such an indirect transfer occurs, the income will be considered to have originated or accrued in India and will be subject to taxation for both non-residents and those who are not typically residents.

Therefore, option (d) is the correct answer

Q9. With reference to the expenditure made by an organization or a company, which of the following statements is/are correct?

1. Acquiring new technology is capital expenditure.

 2. Debt financing is considered capital expenditure, while equity financing is considered revenue expenditure.

 Select the correct answer using the code given below:

 a) 1 only

 b) 2 only

c) Both 1 and 2

 d) Neither 1 nor 2

Show Answer

Answer: a

 Explanation:

• A corporation uses capital expenditures (CapEx) to pay for the purchase, upkeep, and improvement of tangible assets such as real estate, machinery, buildings, plants, and technology. CapEx is frequently utilized by businesses to launch new initiatives or investments. Capital expenditure types can involve investing in real estate, machinery, computers, furnishings, software, and land. Thus, the first statement is true.

• Debt financing is the process by which a business sells debt instruments to individual and/or institutional investors to raise money for working capital or capital expenditures. The people or organizations become creditors in exchange for lending the money and get a guarantee that the principal and the debt's interest will be paid back. Equity financing refers to the alternative method of raising funds in the debt markets through the issuance of stock shares through a public offering. Investments in stocks and debt are both seen as capital expenses. The continuing operating expenses—short-term costs incurred in the course of conducting day-to-day business operations—are known as revenue expenditures. Therefore, assertion 2 is untrue.

Therefore, option (a) is the correct answer.

Q10) With reference to Balance of Payments, which of the following constitutes/ constitute the Current Account ?

 1. Balance of trade

2. Foreign assets

3. Balance of invisibles

4. Special Drawing Rights

Show Answer

Answer: (c)

Explanation

Statements 1 and 3 are correct:

The current account tracks actual transactions, such as import and export goods. The capital account tracks the net balance of international investments. In other words, it keeps track of the flow of money between a nation and its foreign partners.Any deficit in current account is met by equal financial inflows in capital account to keep the BoP in balance.

Q11) The terms 'Marginal Standing Facility Rate' and 'Net Demand and Time Liabilities', sometimes appearing in news, are used in relation to

(a) banking operations

(b) communications networking

(c) military strategies

(d) supply and demand of agricultural products

Show Answer

Answer: a

 Explanation:

• A corporation uses capital expenditures (CapEx) to pay for the purchase, upkeep, and improvement of tangible assets such as real estate, machinery, buildings, plants, and technology. CapEx is frequently utilized by businesses to launch new initiatives or investments. Capital expenditure types can involve investing in real estate, machinery, computers, furnishings, software, and land. Thus, the first statement is true.

• Debt financing is the process by which a business sells debt instruments to individual and/or institutional investors to raise money for working capital or capital expenditures. The people or organizations become creditors in exchange for lending the money and get a guarantee that the principal and the debt's interest will be paid back. Equity financing refers to the alternative method of raising funds in the debt markets through the issuance of stock shares through a public offering. Investments in stocks and debt are both seen as capital expenses. The continuing operating expenses—short-term costs incurred in the course of conducting day-to-day business operations—are known as revenue expenditures. Therefore, assertion 2 is untrue.

Therefore, option (a) is the correct answer.

Q10. With reference to the Indian economy, consider the following statements:

 1. A share of the household's financial savings goes towards government borrowings.

 2. Dated securities issued at market-related rates in auctions form a large component of internal debt.

Which of the above statements are correct?

 a) 1 only

 b) 2 only

 c) Both 1 and 2

 d) Neither 1 nor 2

Show Answer

Answer: c

 Explanation:

• Financial assets accounted for 50.8% of household savings in FY18, with physical assets coming in second at 48.1% and gold/silver ornaments at 1.1%. The single biggest type of financial assets held by households are bank deposits, which are followed by cash, mutual funds, insurance funds, and funds. As a portion of household savings is allocated to government borrowings, with insurance companies and mutual/provident funds being major investors in government securities. Thus, the first statement is true.

• Internal debt includes bonds, compensation, and open-market loans, among other things.

It also covers borrowings made through treasury bills, such as those issued to international financial institutions, commercial banks, and state governments, in addition to non-negotiable, non-interest-bearing dated securities. Thus, assertion 2 is true.

Therefore, option (c) is the correct answer.

Q11. Consider the following statements:

1. Pursuant to the report of H.N. Sanyal Committee, the Contempt of Courts Act, 1971 was passed.

 2. The Constitution of India empowers the Supreme Court and High Court to punish for contempt of themselves.

3. The Constitution of India defines Civil Contempt and Criminal Contempt.

 4. In India, the Parliament is vested with the power to make laws on Contempt of Court.

 Which of the statements given above is/are correct?

 a) 1 and 2 only

 b) 1, 2 and 4

 c) 3 and 4 only

d) 3 only

Show Answer

Answer: b

 Explanation:

• The H. N. Sanyal committee, established in 1961, conducted a thorough analysis of the legislation and issues surrounding contempt of court. The committee's suggestions were mostly adopted. By the committee's report, the 1971 Contempt of Court Act was approved by the legislature. Therefore, statement 1 is true.

• The Indian Constitution's Articles 129 and 215 give the Supreme Court and High Court, respectively, the authority to penalize violators for contempt. Therefore, statement 2 is true.

• The definitions of civil and criminal contempt are found in the Contempt of Courts Act of 1971, not the Constitution. Thus, assertion 3 is untrue.

• As per Article 142 (2), the Supreme Court is empowered to issue any order for the punishment of any contempt of itself, "subject to the provisions of any law made in this behalf by Parliament." Thus, legislation regarding contempt can be made by the Parliament of the Court. Therefore, statement 4 is true.

Therefore, option (b) is the correct answer.

Q14. Consider the following statements:

1. The Constitution of India classifies the ministers into four ranks viz. Cabinet Minister, Minister of State with Independent Charge, Minister of State, and Deputy Minister.

 2. The total number of ministers in the Union Government, including the Prime Minister, shall not exceed 15 percent of the total number of members in the Lok Sabha.

 Which of the statements given above is/are correct?

a) 1 only

b) 2 only

c) Both 1 and 2

 d) Neither 1 nor 2

Show Answer

Explanation:

• Cabinet Ministers, Ministers of State, and Deputy Ministers are the three types of ministers that make up the Council of Ministers. However, the Constitution makes no indication of its dimensions or classification. Its division into three tiers is predicated on the British-developed parliamentary government conventions. It does, however, have legal backing. A "minister" is defined as a "member of the council of ministers, by whatever name called, and includes a deputy minister" by the Salaries and Allowances Act of 1952. Thus, the first assertion is untrue.

• The 91st Constitutional Amendment Act of 2003 has clauses that restrict the number of members in the Council of Ministers. The Central Council of Ministers' entire membership, including the Prime Minister, cannot exceed 15% of the Lok Sabha's total membership. Thus, the second assertion is true.

Therefore, option (b) is the correct answer.

Q15) Consider the following countries :

1. Denmark

2. Japan

3. Russian Federation

4. United Kingdom

5. United States of America

 Which of the above are the members of the 'Arctic Council' ?

 (a) 1, 2 and 3

(b) 2, 3 and 4

 (c) 1, 4 and 5

(d) 1, 3 and 5

Show Answer

Answer: b

 Explanation:

• The consent of both Houses is necessary for all kinds of emergencies. Within a month after its issuance, the declaration of national emergency must get approval from both houses of parliament. President's Rule proclamations require approval from both houses of Parliament within a fortnight after the publication date. Within two months after its issuance, a proclamation proclaiming a financial emergency needs to be ratified by both Houses of Parliament. Therefore, assertion 1 is untrue.

• It is only possible to move a motion of no confidence in the Lok Sabha, not the Rajya Sabha. Any member of the Lok Sabha may move it. The steps involved in proposing a no-confidence resolution are outlined in Rule 198 of the Lok Sabha's Rules of Procedure and Conduct. Thus, assertion 2 is accurate.

• Either House of Parliament may bring allegations of impeachment against the President.

The President is removed from office as of the day the impeachment resolution is enacted if the other House likewise upholds the charges and votes in favor of it with a majority of two-thirds of the members present. Therefore, assertion 3 is untrue.

Therefore, option (b) is the correct answer.

Q16. With reference to anti-defection law in India, consider the following statements:

 1. The law specifies that a nominated legislator cannot join any political party within six months of being appointed to the House.

 2. The law does not provide any time frame within which the presiding officer has to decide a defection case.

 Which of the statements given above is/are correct?

 a) 1 only

 b) 2 only

c) Both 1 and 2

d) Neither 1 nor 2

Show Answer

Answer: b

 Explanation:

• A nominated member of a House loses his eligibility to serve as a member of the House if he joins a political party after six months after taking up his seat. This is by the Anti-defection Law. He or she can therefore join a political party six months after signing up for membership expiration. Therefore, assertion 1 is untrue.

• The House Presiding Officer shall decide any question about disqualification resulting from defection. There is no deadline set by law for the presiding officer to decide a defection case. Therefore, statement 2 is true.

Therefore, option (b) is the correct answer.

Q17. Consider the following statements:

1. The Attorney General of India and Solicitor General of India are the only officers of the Government who are allowed to participate in the meetings of the Parliament of India.

 2. According to the Constitution of India, the Attorney General of India submits his resignation when the Government which appointed him resigns.

Which of the statements given above is/are correct?

a) 1 only

 b) 2 only

c) Both 1 and 2

d) Neither 1 nor 2

Show Answer

Answer: d

Explanation:

• The Attorney General has the right to audience in all Indian courts while carrying out his official duties. Additionally, by Article 88, he is entitled to speak and participate in any joint sitting of the Houses of Parliament as well as any he may be named to a committee of the Parliament, but he will not have voting rights.

There is no such privilege for the Solicitor General to attend parliamentary sessions. Thus, the first statement is untrue.

• The Attorney General serves as long as the President so desires. This implies that the president has the authority to remove him at any moment. He can also tender his resignation to the president to leave his position. Traditionally, he steps down when the council of ministers is removed or resigns because he was appointed against its advice. As a result, it is customary and unmentioned in the Indian Constitution. Therefore, assertion 2 is untrue.

 Therefore, option (d) is the correct answer.

Q18. With reference to the writs issued by the Courts in India, consider the following statements:

1. Mandamus will not lie against a private organization unless it is entrusted with a public duty.

 2. Mandamus will not lie against a Company even though it may be a Government Company.

 3. Any public-minded person can be a petitioner to move the Court to obtain the writ of Quo Warranto.

Which of the statements given above are correct?

 a) 1 and 2 only

 b) 2 and 3 only

 c) 1 and 3 only

d) 1, 2 and 3

Show Answer

Answer: c

 Explanation:

• In the case of Binny Ltd. and Anr. v. V. Sadasivan and Ors, the Supreme Court ruled that although a writ of mandamus can be used to enforce a purely private contract between parties, it must also contain a public law component. The extent of Mandamus is not based on the identification of the authority being sought, but rather on the nature of the duty to be imposed. The public law remedy may be used if the private entity is carrying out a public function and any right is being denied in connection with the public duty placed upon it. Therefore, statement 1 is true.

• In literal terms, mandamus means "we command." It is an order from the court to a public official that he must carry out his official responsibilities that he has neglected or declined to do. It may be filed against the government, a corporation, a lower court, a tribunal, or any other public body business with the same objective. Thus, statement 2 is untrue.

• the individual who is not necessarily the offended party may seek a Quo-Warranto writ. Therefore, statement 3 is true.

Therefore, option (c) is the correct answer.

Q19. With reference to Ayushman Bharat Digital Mission, consider the following statements:

 1. Private and public hospitals must adopt it.

 2. As it aims to achieve universal health coverage, every citizen of India should be part of it ultimately.

 3. It has seamless portability across the country.

 Which of the statements given above is/are correct?

a) 1 and 2 only

 b) 3 only

c) 1 and 3 only

d) 1, 2 and 3

Show Answer

Answer: b

Explanation:

• A healthcare facility's or institutions voluntary participation in the Ayushman Bharat Digital Mission (ABDM) will be decided by the relevant management (private or public). Therefore, assertion 1 is untrue.

• Its goal is to establish a nationwide digital health ecosystem that supports universal health care in a way that is effective, inexpensive, inclusive, fast, and secure. It does this by offering a variety of data, information, and infrastructure services and by appropriately utilizing open, standards-based, digital systems that are interoperable and guarantee the security, privacy, and confidentiality of personal health information. However, obtaining an Ayushman Bharat Health Account (ABHA) Number is not required of all citizens. An ABHA Number may be created by anyone who wants to take part in the Ayushman Bharat Digital Mission and have their records accessible online. Therefore, assertion 2 is untrue.

• A key component of the Ayushman Bharat Digital Mission (ABDM) will be the implementation of Health Information Standards. The ABDM will have a nationwide reach and facilitate smooth mobility throughout the nation via the use of a Health ID, or Personal Health Identifier in terms of national portability. Therefore, statement 3 is true.

Therefore, option (b) is the correct answer.

Q20. With reference to the Deputy Speaker of Lok Sabha, consider the following statements:

 1. As per the Rules of Procedure and Conduct of Business in Lok Sabha, the election of Deputy Speaker shall be held on such date as the Speaker may fix.

 2. There is a mandatory provision that the election of a candidate as Deputy Speaker of Lok Sabha shall be from either the principal opposition party or the ruling party.

3. The Deputy Speaker has the same power as the Speaker when presiding over the sitting of the House and no appeal lies against his rulings.

 4. The well-established parliamentary practice regarding the appointment of a Deputy Speaker is that the motion is moved by the Speaker and duly seconded by the Prime Minister.

 Which of the statements given above are correct?

a) 1 and 3 only

b) 1, 2 and 3

c) 3 and 4 only

d) 2 and 4 only

Show Answer

Answer: a

 Explanation:

• The Rules of Procedure and Conduct of Business in the Lok Sabha, Rule 8, governs the election of the deputy speaker. The election "shall be held on such date as the Speaker may fix," per the Rule, and the Deputy Speaker is chosen following a resolution suggesting his name has been abandoned. Therefore, statement 1 is true.

• The Speaker and Deputy Speaker were typically members of the ruling party till the 10th Lok Sabha. There has been general agreement since the 11th Lok Sabha that the leading opposition party holds the office of Deputy Speaker, while the Speaker is chosen by the ruling party or alliance. Therefore, assertion 2 is untrue.

• "The Deputy Speaker shall perform the duties of the office while the Speaker's seat is vacant," states Article 95(1). When leading a House session, the Deputy Speaker is endowed with the same authority as the Speaker. Every mention of the Speaker in the Rules is considered to be an allusion to the Deputy Speaker during his chairmanship. The Deputy Speaker or any other person presiding over a House session in the Speaker's absence may not be appealed to the Speaker. Hence, statement 3 is true. ●

• A motion is introduced by the Parliamentary Affairs Minister and properly seconded by either the opposition or the Treasury benches. The motion is then carried by the House according to established parliamentary procedure. Thus, statement 4 is untrue.

Therefore, option (a) is the correct answer.

Q21. Among the following crops, which one is the most important anthropogenic source of both methane and nitrous oxide?

a) Cotton

b) Rice

c) Sugarcane

d) Wheat

Show Answer

Answer: b

 Explanation:

• Agriculture is estimated to account for 10%–20% of anthropogenic greenhouse gas (GHG) emissions worldwide. In 2005, it accounted for 50% and 60% of the total anthropogenic methane (CH4) and nitrous oxide (N2O) emissions, respectively. Rice paddies are considered one of the most important sources of methane and nitrous oxide emissions, which have attracted considerable attention due to their contribution to global warming.

Therefore, option (b) is the correct answer.

Q22 “System of Rice intensification” of cultivation, in which alternate wetting and drying of rice field is practised, results in:

 1. Reduced seed requirement

 2. Reduced methane production

 3. Reduced electricity consumption

 Select the correct answer using the code given below:

a) 1 and 2 only

b) 2 and 3 only

c) 1 and 3 only

d) 1, 2 and 3

Show Answer

Answer: d

Explanation:

• To cultivate rice using the System of Rice Intensification, as much organic manure as possible must be used. Young seedlings are first planted singly at a greater spacing in a square pattern, and intermittent irrigation is used to keep the soil moist but not saturated. Frequent interplanting cultivation using an actively aerating weed eater.

With the use of the agroecological and climate-smart System of Rice Intensification (SRI), farmers can enhance yields by 30–50% or more and use 90% less seed. Thus, point 1 is accurate.

• Reduce irrigation water usage by 30–50% (with AWD and other techniques) while using less electricity. Point 3 is thus accurate.

• Utilize 30–100% fewer chemical pesticides and fertilizers

• A decrease in the emissions of methane. Point 2 is thus accurate.

Therefore, option (d) is the correct answer.

Q23 Which one of the following lakes of West Africa has become dry and turned into desert?

a) Lake Victoria

b) Lake Faguibine

c) Lake Oguta

d) Lake Volta

Show Answer

Answer: b

 Explanation:

• Since the 1970s, Lake Faguibine in northern Mali has been dry.

• The Lake Faguibine System consists of four connected lakes.

• Historically, one of Mali's most productive regions was located 80 km west of Timbuktu. However, the lakes were dried up over seven years due to droughts in the 1970s. Sand then covered the passages that connected the lakes to the River Niger, making it impossible for water to reach the lakes when rain eventually returned. The wealth of the area vanished with the flood.

Therefore, option (b) is the correct answer.

Q24. Gandikota Canyon of South India was created by which one of the following rivers?

 a) Cauvery

 b) Manjira

c) Pennar

d) Tungabhadra

Show Answer

Answer: c

Explanation:

• The Andhra Pradesh district of Kadappa has the Gandikota Canyon. The gorge is a breathtaking jumble of sharp rocks arranged in crimson layers. The waters of the renowned Pennar river, which rises in the Erramala highlands, have carved out this breathtaking gorge. The region is currently referred to as the Grand Canyon of India because of its similarities with the well-known US monument. A 12th-century fort perches atop the hillsides encircling the gorge, along with two other historic temples, can be found there.

Therefore, option (c) is the correct answer.

Q25. Consider the following pairs:

Sl. No. Peak Mountains

1. Namcha Barwa : Garhwal Himalaya

2. Nanda Devi : Kumaon Himalaya

3. Nokrek : Sikkim Himalaya

Which of the pairs given above is/are correctly matched?

 a) 1 and 2

b) 2 only

c) 1 and 3

d) 3 only

Show Answer


Answer b

Explanation:

• In Arunachal Pradesh, Namcha Barwa is situated close to the easternmost Himalayas. The Namcha Barwa is the point where the powerful Brahmaputra River enters India. It is not found in Himalayan Garhwal. Therefore, pair 1 is not matched correctly.

• The Kumaon Himalayas stretch 320 km between the Sutlej and Kali rivers. Important peaks in this area include Nanda Devi (7,817 m), Kamet (7,756 m), Trisul (7,140 m), Badrinath (7,138 m), Kedarnath (6,968 m), and Gogotri (6,510 m). Pair 2 is thus appropriately matched.

• The highest peak in Meghalaya's Garo Hills range is Nokrek. At 1412 meters above sea level, it is the highest point in the Garo Hills region and the source of most of the major rivers and streams. Pair 3 is therefore incorrect.

Therefore, option (b) is the correct answer.

Q26. The term “Levant” often heard in the news roughly corresponds to which of the following regions?

a) Region along the eastern Mediterranean shores

b) Region along North African shores stretching from Egypt to Morocco

c) Region along the Persian Gulf and the Horn of Africa

d) The entire coastal areas of the Mediterranean Sea

Show Answer

Answer: a

 Explanation:

• Levant, historically, referred to as the area around the eastern Mediterranean beaches, is derived from the French lever, which means "to rise," as in sunrise, denoting the east representing what is now known as Syria, Israel, Jordan, Lebanon, and several surrounding regions.

Therefore, option (a) is the correct answer.

Q27. Consider the following countries:

1. Azerbaijan

2. Kyrgyzstan

3. Tajikistan

4. Turkmenistan

5. Uzbekistan

 Which of the above have borders with Afghanistan?

a) 1, 2 and 5 only

b) 1, 2, 3 and 4 only

c) 3, 4 and 5 only

d) 1, 2, 3, 4 and 5

Show Answer

Answer: c

Explanation:

• Afghanistan shares borders with Pakistan to the east and south, Iran to the west, and Turkmenistan, Uzbekistan, and Tajikistan in Central Asia to the north.

Therefore, option (c) is the correct answer.

Q28. With reference to India, consider the following statements:

1. Monazite is a source of rare earths.

2. Monazite contains thorium.

3. Monazite occurs naturally in the entire Indian coastal sands in India.

4. In India, Government bodies only can process or export monazite.

 Which of the statements given above are correct?

 a) 1, 2 and 3 only

 b) 1, 2 and 4 only

c) 3 and 4 only

d) 1, 2, 3 and 4

Show Answer

Answer: (b)

 Explanation:

One of the rare earth elements found in beach sand is monazite, which includes lanthanum, cerium, praseodymium, neodymium, and other elements. In general, monazite includes between 55 and 60 percent rare earth oxide. Monazite also contains thorium. Consequently, assertions 1 and 2 are true.

Q29. In the northern hemisphere, the longest day of the year normally occurs in the:

a) First half of the month of June

b) Second half of the month of June

c) First half of the month of July

d) Second half of the month of July

Show Answer

Answer: b

Explanation:

• June 21 is the longest day in 2021 for people who live north of the equator. Technically speaking, this day is known as the summer solstice, which is the longest day of the summer. That happens when the sun is exactly overhead the Tropic of Cancer, or more precisely when it is directly over 23.5 degrees latitude north. On Monday, this will occur at approximately 9:02 a.m. (Indian Standard Time).

• The Earth's axis, around which the planet rotates once every day, is tilted so that the North Pole is oriented toward the sun and the South Pole is oriented away from it during the solstice.

Therefore, option (b) is the correct answer.

Q30. Consider the following pairs:

Sl. No. Wetland/Lake Location
1 Hokera Wetland : Punjab
2 Renuka Wetland : Himachal Pradesh
3 Rudrasagar Lake : Tripura
4 Sasthamkotta Lake : Tamil Nadu

How many pairs given above are correctly matched?

a) Only one pair

b) Only two pairs

c) Only three pairs

d) All four pairs

Show Answer

Answer: b

Explanation:

• The Hokera Wetland is situated in Jammu and Kashmir's Union Territory. It is situated behind the snow-covered Pir Panchal in the northwest Himalayan biogeopgraphic province of Kashmir. Therefore, pair 1 is not matched correctly.

• Himachal Pradesh is home to Renuka Wetand. It is a naturally occurring marsh that is nourished by a little stream that runs from the lower Himalayan to the Giri river. It features freshwater springs and inland subterranean karst structures. Pair 2 is thus appropriately matched.

• The location of Rudrasagar Lake is in Tripura. Located in the northeastern highlands, this lowland sedimentation reservoir is supplied by three year-round streams that empty into the River Gomti. Pair 3 is thus appropriately matched.

• Kerala is where Sasthamkotta Lake is situated. It is the largest freshwater lake in Kerala state, which is located in the southwest of the nation. It is supplied by springs and provides drinking water to 500,000 people in the Kollam district. Pair 4 is therefore not well matched.

As a result, the right response is option (b).

Q31. Consider the following:

1. Aarogya Setu

2. COWIN

3.DigiLocker

4. DIKSHA

Which of the above are built on top of open-source digital platforms?

 a) 1 and 2 only

b) 2, 3 and 4 only

c) 1, 3 and 4 only

d) 1, 2, 3 and 4

Show Answer

Answer: d

Explanation:

Software that has its source code available for anyone to see, alter, and improve is known as open-source software. A "program" or "application"'s "source code" is the portion of the software that most users never see, but computer programmers can alter to alter how a piece of software functions. When a computer program's source code is available to programmers, they can enhance it by adding new features or correcting malfunctioning areas. Open-source digital platforms underpin the development of Arogya Setu, COWIN, Digilocker, and DIKSHA.

As a result, choice (d) is the right response.

Q32. With reference to Web 3-0, consider the following statements:

      1. Web 3.0 technology enables people to control their own data.

      2. In Web 3.0 world, there can be blockchain based social networks.

      3. Web 3.0 is operated by users collectively rather than a corporation.

     Which of the statements given above are correct?

     a) 1 and 2 only

     b) 2 and 3 only

     c) 1 and 3 only

     d) 1, 2 and 3

Show Answer

Answer: d

     Explanation:

• The third generation of the internet, known as Web 3.0, has the potential to be just as disruptive and significant a paradigm change as Web 2.0. The fundamental ideas of Web 3.0 are better user utility, openness, and decentralization.

• Below are some of Web3.0's features: A fundamental principle of Web 3.0 is decentralization. In Web 2.0, computers locate data that is kept at a fixed location, usually on a single server, using HTTP in the form of distinct web addresses. With Web 3.0, data might be stored in several places at once and be decentralized because it would be found based on its content. Users would have more influence as a result of this dismantling the enormous datasets that are currently controlled by internet behemoths like Google and Meta. Web 3.0 has made the data produced by various and more

Users will sell powerful computing resources through decentralized data networks, guaranteeing that they maintain ownership control. Examples of these resources include smartphones, desktop computers, appliances, cars, and sensors. Thus, claims 1 and 3 are true.

• Web 3.0 will be decentralized, run on open-source software, and enable direct interaction between users without the need for a trusted middleman. Anyone can join without a governing body's permission. Web 3.0 apps will therefore operate on decentralized peer-to-peer networks or blockchains. Therefore, statement 2 is true.

   Consequently, choice (d) is the appropriate answer.

Q33. With reference to "Software as a Service (SaaS)", consider the following statements:

 1. SaaS buyers can customise the user interface and can change data fields.

 2. SaaS users can access their data through their mobile devices.

 3. Outlook, Hotmail and Yahoo! Mail are forms of SaaS.

 Which of the statements given above are correct?

a) 1 and 2 only

b) 2 and 3 only

c) 1 and 3 only

d) 1, 2 and 3

Show Answer

Answer: d

Explanation:

In the software distribution model known as "software as a service" (SaaS), apps are hosted by cloud providers and made accessible to end users via the internet. According to this arrangement, an independent software vendor (ISV) can hire a different cloud provider to house the software-on-demand (SaaS):

• In this model, the provider provides network-based access to a single copy of a program that was developed especially for SaaS distribution. Every customer has access to the same source code for the program, and new features and functionality are rolled out to everyone at the same time.

• The customer's data for each model may be stored locally, in the cloud, or both locally and in the cloud, depending on the service-level agreement (SLA). Application programming interfaces (APIs) allow businesses to combine SaaS apps with other software. For instance, a company can create its software tools and link them with the SaaS service by using the APIs provided by the SaaS provider.

• SaaS purchasers can therefore alter data fields and personalize the user interface. SaaS subscribers can now access their data on mobile devices thanks to this as well. Consequently, both claims 1 and 2 are true.

• Email services, offered by businesses like Yahoo, Hotmail, and Outlook, are another type of Software as a Service (SAAS). Therefore, statement 3 is true.

As a result, choice (d) is the right response.

Q34. Which one of the following statements best reflects the idea behind the "Fractional Orbital Bombardment System" often talked about in media?

a) A hypersonic missile is launched into space to counter the asteroid approaching the Earth and explode it in space.

b) A spacecraft lands on another planet after making several orbital motions.

c) A missile is put into a stable orbit around the Earth and deorbits over a target on the Earth.

d) A spacecraft moves along a comet with the same speed and places a probe on its surface.

Show Answer

Answer: b

Explanation:

• June 21 is the longest day in 2021 for people who live north of the equator. Technically speaking, this day is known as the summer solstice, which is the longest day of the summer. That happens when the sun is exactly overhead the Tropic of Cancer, or more precisely when it is directly over 23.5 degrees latitude north. On Monday, this will occur at approximately 9:02 a.m. (Indian Standard Time).

• The Earth's axis, around which the planet rotates once every day, is tilted so that the North Pole is oriented toward the sun and the South Pole is oriented away from it during the solstice.

Therefore, option (b) is the correct answer.

Q29. In the northern hemisphere, the longest day of the year normally occurs in the:

a) First half of the month of June

b) Second half of the month of June

c) First half of the month of July

d) Second half of the month of July

Show Answer

Answer: c

 Explanation:

• The Fractional Orbital Bombardment System (FOBS) aims to deorbit a weapon over the target and place it in a stable orbit. The warhead will complete a circle if it is aimed at the target and launched in the proper alignment. Naturally, with the earth in motion Although it is more of a spiral than a ring during this, the warhead still crosses the target because the horizontal movement is opposed. The main idea is that a missile can be shot in one direction and travel the full circumference of the earth to reach its objective, or it can be flown directly to the target.

Option (c) is the right response as a result.

Q35. Which one of the following is the context in which the term “Qubit” is mentioned?

a) Cloud Services

b) Quantum computing

c)Visible light communication technologies

d) Wireless Communication Technologies

Show Answer

Answer: b

Explanation:

• The quantum mechanical counterpart of a classical bit is called a qubit, or quantum bit. Information is represented in bits in classical computing, where a bit might have a value of zero or one.

• Qubits are used in quantum computing to store and encode information. Qubits have two possible states: 0 and 1 or in a linear combination of both states (in contrast to a classical bit). This phenomenon is known as superposition.

Option (b) is the right response as a result.

Q36. Consider the following communication Technologies:

 1. Closed- Circuit Television

2. Radio Frequency Identification

3. Wireless Local area Network

Which of the above are considered short-range devices/technologies?

a) 1 and 2 only

b) 2 and 3 only

c) 1 and 3 only

d) 1, 2 and 3

Show Answer

Answer: d

Explanation:

• Due to their low broadcast power and hence short range, short-range devices (SRD) are radio devices with minimal chance of interfering with other radio services. A "short range device" is any of several kinds of wireless technology, including several kinds of :

• Alarms and movement detectors

• Closed-circuit television (CCTV)

• Access control (including door and gate openers)

• Cordless audio devices, including wireless microphones

• Industrial control

• Local Area Networks

• Implants for medical purposes

• Metering apparatus

• Identification using radio frequency (RFID)

• Remote control

• Telematics for Road Transport

• Telemetry.

As a result, choice (d) is the right response.

Q37.Consider the following statements:

 1. Biofilms can form on medical implants within human tissues.

 2. Biofilms can form on food and food processing surfaces.

 3. Biofilms can exhibit antibiotic resistance.

 Which of the statements given above are correct?

a) 1 and 2 only

b) 2 and 3 only

c) 1 and 3 only

d) 1, 2 and 3

Show Answer

Answer: d

 Explanation:

• A group of one or more types of microorganisms that can grow on a variety of surfaces is known as a biofilm. Biofilms are formed by bacteria, fungus, and protists, among other microorganisms.

• One typical illustration of a biofilm dental plaque is a sticky bacterial accumulation that develops on the teeth's surfaces. Pond scum is an additional instance. It has been discovered that biofilms grow on metals and minerals. They have been discovered above, below, and underwater.

• They can develop on the tissues of plants and animals, on surfaces used in food preparation and consumption, and on implanted medical devices like pacemakers and catheters. Thus, both statements 1 and 2 are true.

• There are benefits to living inside a biofilm for bacteria. They are observed to be resistant to antibiotics.

Therefore, statement 3 is true.

Q38. Consider the following statements in respect of probiotics:

1. Probiotics are made both of bacteria and yeast.

2. The organisms in probiotics are found in foods we ingest but they do not naturally occur in our gut.

3. Probiotics help in the digestion of milk sugars.

 Which of the statements given above is/are correct?

a) 1 only

b) 2 only

c) 1 and 3

d) 2 and 3

Show Answer

Answer: c

Explanation:

• Probiotics are living microorganisms that are administered or taken to improve health. They are present in nutritional supplements, yogurt other fermented meals, and cosmetics. Even though bacteria and other microorganisms are beneficial, despite being perceived as dangerous "germs." Certain bacteria aid in vitamin production, disease-causing cell destruction, and food digestion.

• Numerous types of bacteria can be found in probiotics. The most prevalent types of bacteria are those that fall under the Lactobacillus and Bifidobacterium genera. Probiotics can also be made from other bacteria or yeasts, such as Saccharomyces boulardii. A large number of the microbes in Probiotic supplements are identical to or comparable to the microorganisms found naturally in our systems. Accordingly, claim 1 is true while claim 2 is false.

• The probiotic bacteria Lactobacillus acidophilus is found in the human intestine and other regions of the body by nature. This bacterium aids in the digestive system's conversion of carbohydrates into lactic acid, including lactose. Therefore, statement 3 is true.

Option (c) is the right response as a result.

Q39. In the context of vaccines manufactured to prevent COVID-19 pandemic, consider the following statements:

1. The Serum Institute of India produced COVID-19 vaccine named Covishield using mRNA platform.

2. Sputnik V vaccine is manufactured using vector based platform.

3. COVAXIN is an inactivated pathogen based vaccine.

 Which of the statements given above are correct?

a) 1 and 2 only

b) 2 and 3 only

c) 1 and 3 only

d) 1, 2 and 3

Show Answer

Answer: b

Explanation:

The SARS-CoV-2 Spike (S) glycoprotein is encoded by the recombinant, replication-deficient chimpanzee adenovirus vector Covishield, which is manufactured by the Serum Institute of India. After administration, a portion of the corona virus's genetic material is expressed, which causes an autoimmune reaction. Therefore, assertion 1 is untrue.

• The Gamaleya Institute in Moscow develops the Sputnik V vaccination. The non-replicating human adenovirus vector platform serves as its foundation. Therefore, statement 2 is true.

• A whole-virion inactivated coronavirus vaccine called COVAXIN is also used to make vaccinations against hepatitis A, rabies, and influenza. The company that makes it is Bharat Biotech. Thus, assertion 3 is true.

• The messenger RNAs in mRNA vaccines cause our bodies to mount an immunological response, which results in the production of antibodies and shields us against the virus. Researchers at the CSIR-Centre for Cellular and Molecular Biology in India are now evaluating a potential mRNA vaccination against Covid-19.

Option (b) is the right response as a result.

Q40. If a major solar storm (solar flare) reaches the Earth, which of the following are the possible effects on the Earth?

 1. GPS and navigation systems could fail.

 2. Tsunamis could occur at equatorial regions.

 3. Power grids could be damaged.

4. Intense auroras could occur over much of the Earth.

5. Forest fires could take place over much of the planet.

6. Orbits of the satellites could be disturbed.

 7. Shortwave radio communication of the aircraft flying over polar regions could be interrupted.

 Select the correct answer using the code given below:

a) 1, 2, 4 and 5 only

b) 2, 3, 5, 6 and 7 only

c) 1, 3, 4, 6 and 7 only

d) 1, 2, 3, 4, 5, 6 and 7

Show Answer

Answer: c

 Explanation:

Large explosions from the sun's surface that release powerful bursts of electromagnetic radiation are known as solar flares. Which category the flare falls into depends on how strong the explosion is. Flares of the X-class are the strongest, followed by those of the M-, C-, and B-class; A-class. The smallest are flares.

• When magnetic energy accumulates in the solar atmosphere and is abruptly released, solar flares take place. The solar cycle, an approximately 11-year pattern of solar activity fueled by the sun's magnetic field, is inextricably tied to these eruptions.

• Flares of various kinds, especially X-class flares, have an impact on Earth, satellites, and even people on board.

• They might harm the world's electricity grids and cause GPS and navigation systems to malfunction. A lot of the Earth might experience strong auroras as a result of them. strong solar Flares may also interfere with space operations. These could cause disruptions to satellite orbits and shortwave radio communication breakdowns for airplanes operating over polar regions. Points 1, 3, 4, 6, and 7 are therefore accurate.

• It should be mentioned that as the solar wind penetrates more into the topography, it becomes weaker and is directed towards Earth. Therefore, forest fires and tsunamis might not be caused by solar flares. Therefore, points 2 and 5 are untrue.

Option (c) is the right response as a result.

Q41. "Climate Action Tracker" which monitors the emission reduction Pledges of different countries is a:

 a) Database created by coalition of research organisations

 b) Wing of ‘International Panel of Climate Change”

 c) Committee under “United Nations Framework Convention on Climate Change”

 d) Agency promoted and financed by United Nations Environment Programme and World Bank

Show Answer

Answer: a

 Explanation:

The goal of the Paris Agreement, which was reached globally, is to "hold warming well below 2°C, and pursue efforts to limit warming to 1.5°C." The Climate Action Tracker is an impartial scientific analysis that monitors government climate action and measures it against this goal. Two people working together Since 2009, the Climate Analytics and New Climate Institute organizations have been the sources of this impartial analysis for policymakers on behalf of the CAT.

Option (a) is the right response as a result.

Q42. Consider the following statements:

1. “The Climate Group” is an international non-profit organization that drives climate action by building large networks and runs them.

2. The International Energy Agency in partnership with the Climate Group launched a global initiative “EP100”.

3. EP100 brings together leading companies committed to driving innovation in energy efficiency and increasing competitiveness while delivering on emission reduction goals.

4. Some Indian companies are members of EP100.

5. The International Energy Agency is the Secretariat to the “Under2 Coalition”.

 Which of the statements given above are correct?

a) 1, 2, 4 and 5

b) 1, 3 and 4 only

c) 2, 3 and 5 only

d) 1, 2, 3, 4 and 5

Show Answer

Answer: b

 Explanation:

• Founded in 2003, The Climate Group is a global nonprofit organization. By 2050, it hopes to have achieved net zero carbon emissions worldwide and increased prosperity for all. It creates and maintains networks. It unites organizations to unleash the potential of group action that has similar goals and exerts influence. Therefore, statement 1 is true.

• The global campaign EP100 is spearheaded by the non-profit organization Climate Group. The International Energy Agency did not start it. Therefore, assertion 2 is untrue.

• Over 120 energy-smart companies that are dedicated to tracking and reporting on increases in energy efficiency are brought together by EP100. By implementing energy efficiency in boardrooms as well as boiler rooms, members are cutting emissions while boosting their competitiveness and encouraging others to take their example. Therefore, statement 3 is true.

• Members of EP100 include numerous Indian businesses, including UltraTech Cement Limited, Godrej Industries Ltd., JSW Cement Ltd., and Mahindra & Mahindra Ltd. Therefore, statement 4 is true.

• More than 270 governments, representing 1.75 billion people and 50% of the world economy, are members of the Under2 Coalition. Its members pledge to limit the rise in global temperatures well below 2°C, aiming for 1.5°C. The Under2 Coalition's secretariat is called Climate Group.

Therefore, assertion 5 is untrue.

Consequently, choice (b) is the right response as a result.

Q43. “If rainforests and tropical forests are the lungs of the Earth, then surely wetlands function as its kidneys.” Which one of the following functions of wetlands best reflects the above statement?

 a) The water cycle in wetlands involves surface runoff, subsoil percolation and evaporation.

 b) Algae form the nutrient base upon which fish, crustaceans, molluscs, birds, reptiles and mammals thrive.

c) Wetlands play a vital role in maintaining sedimentation balance and Soil stabilization.

d) Aquatic plants absorb heavy metals and excess nutrients.

Show Answer

Answer: d

 Explanation:

• As to the Ramsar Convention, wetlands encompass regions of marsh, fen, peatland, or water, regardless of their natural or manmade origin, duration, or state, containing fresh, brackish, or salt water that is either static or moving. They also include marine water areas where the low tide depth does not surpass six feet.

• Wetlands provide crucial ecological services like water regulation, including flood control and water purification, which are necessary for humans, other ecosystems, and our climate. Wetlands are sometimes referred to as the "Kidneys of the Earth" because they also absorb carbon dioxide, which helps to slow down global warming and minimize pollution.

As a result, choice (d) is the right response.

Q44. In the context of WHO Air Quality Guidelines, consider the following statements:

1. The 24-hour mean of PM2.5 should not exceed 15 µg/m3 and annual mean of PM2.5 should not exceed 5 µg/m3 .

 2. In a year, the highest levels of ozone pollution occur during the periods of inclement weather.

 3. PM10 can penetrate the lung barrier and enter the bloodstream.

 4. Excessive ozone in the air can trigger asthma.

 Which of the statements given above are correct?

a) 1, 3 and 4

 b) 1 and 4 only

 c) 2, 3 and 4

d) 1 and 2 only

Show Answer

Answer: b

 Explanation:

The WHO Air Quality Guidelines state that 24-hour average exposures to PM2.5 should not exceed 15 µg/m3 more than three or four days per year, and that annual average concentrations of PM2.5 should not exceed 5 µg/m3. Therefore, statement 1 is true.

In hot, sunny weather, certain pollutants, such ground-level ozone, are produced more effectively.

• Sunlight is necessary for the processes that produce the dangerous ozone in our atmosphere. In cities and neighboring rural regions, ozone levels can reach deadly levels during the summer, particularly during strong heat waves. Lowering ozone pollution can be aided by humidity. Ozone formation slows down during the day due to afternoon thunderstorm clouds blocking sunlight, and the moisture from the storm destroys any ozone that has already created. Thus, statement 2 is untrue.

• According to studies, particulates with a diameter of 10 μm (PM10) can get into the circulation but not the lungs. PM2.5 particles, which have a diameter of 2.5 μm or less, have the ability to enter the bloodstream, travel to the distal lung segments, including the alveoli, and can

permeating blood-organ barriers and impacting several organ systems, including the kidney, liver, heart, and brain. Thus, assertion 3 is untrue.

• When sunlight combines with hydrocarbon and nitrogen oxide emissions from motor vehicles and other sources, it creates ozone pollution. It worsens chest tightness, wheezing, and coughing, especially in vulnerable kids who play outside in contaminated surroundings. Because too much ozone is highly irritating to the lungs and airways, it can aggravate asthma. Therefore, statement 4 is true.

Option (b) is the right response as a result.

Q45. With reference to “Gucchi” sometimes mentioned in the news, consider the following statements: 1. It is a fungus.

2. It grows in some Himalayan forest areas.

 3. It is commercially cultivated in the Himalayan foothills of north-eastern India.

Which of the statements given above is/are correct?

 a) 1 only

b) 3 only

 c) 1 and 2

d) 2 and 3

Show Answer

Answer: c

 Explanation:

The most expensive mushrooms in the world are gucchi, which are grown in the Himalayan foothills. Therefore, statement 1 is true.

• They can be found growing in temperate conifer woods and the chilly foothills of the Himalayas.

• Jammu & Kashmir, Uttaranchal, and Himachal Pradesh are the best places for them. Rainfall is ideal for their growth, but it may take months for enough to be gathered, dried, and sold. The slopes of the Himalayas in northeastern India are not used for their commercial cultivation. As a result, assertion 2 is true but statement 3 is false.

Option (c) is the right response as a result.

Q46. With reference to polyethylene terephthalate, the use of which is so widespread in our daily lives, consider the following statements:

 1. Its fibres can be blended with wool and cotton fibres to reinforce their properties.

 2. Containers made of it can be used to store any alcoholic beverage.

 3. Bottles made of it can be recycled into other products.

 4. Articles made of it can be easily disposed of by incineration without causing greenhouse gas emissions.

 Which of the statements given above are correct?

a) 1 and 3

b) 2 and 4

c) 1 and 4

d) 2 and 3

Show Answer

Answer: a

Explanation:

When it comes to polyester, polyethylene terephthalate (PET) is the most commonly utilized material. They are frequently employed in mixes of durable materials like cotton, rayon, and wool to strengthen the natural qualities of those materials and improve the fabric's ability to to get over wrinkles. Thus, assertion 1 is true.

• The water penetration is not ideal, especially for highly concentrated spirits in tiny PET bottles and high storage temperatures. In reality, if the temperature rises while the alcohol is being stored, the leaching effects of plastic and plastic chemicals (such ethylene glycol) tend to seep extensively into the alcohol. Therefore, assertion 2 is untrue.

• PET bottles are recyclable and can be made again into films, fibers, bottles, thermoformed packaging, and strapping. Therefore, statement 3 is true.

When plastics burn, harmful chemicals include furans, dioxins, mercury, and polychlorinated biphenyls, or BCPs, into the atmosphere, endangering both human and animal health as well as plants. Thus, statement 4 is untrue.

Option (a) is the right response as a result.

Q47. Which of the following is not a bird?

a) Golden Mahseer

 b) Indian Nightjar

c) Spoonbill

d) White Ibis

Show Answer

Answer: a

 Explanation:

• One species of cyprinid fish is the golden mahseer. Mahseer, which means "tiger" amid fish, is generally translated as "mahi" (fish) and "sher" (tiger). It is a huge cyprinid that is regarded as the most resilient freshwater sport fish.

Option (a) is the right response as a result.

Q48. Which of the following are nitrogen-fixing plants?

1. Alfalfa

2. Amaranth

3. Chickpea

4. Clover

5. Purslane (Kulfa)

6. Spinach

Select the correct answer using the code given below:

 a) 1, 3 and 4 only

b) 1, 3, 5 and 6 only

c) 2, 4, 5 and 6 only

d) 1, 2, 4, 5 and 6

Show Answer

Answer: a

Explanation:

• Alfalfa's symbiotic nitrogen fixation process supplies significant amounts of nitrogen (N) to soil organic matter, succeeding crops, and livestock operations. Thus, point 1 is accurate.

• Chickpeas produce root nodules that house symbiotic Nfixing bacteria and facilitate biological N fixation (BNF). Thus, point three is accurate.

• Through a symbiotic interaction with Rhizobium bacteria that infects the plant's roots, nitrogen is "fixed" in clovers. The bacteria supply the "machinery" required to transform air nitrogen into a form that plants can use, while the plant supplies the energy for the bacteria to do so. Point 4 is thus accurate.

• Spinach, amaranth, and purslane (Kulfa) are not plants that fix nitrogen. Thus, the points 2, 5, and 6 are false.

Q49. “Biorock technology” is talked about in which one of the following situations?

 a) Restoration of damaged coral reefs

b) Development of building materials using plant residues

c) Identification of areas for exploration/extraction of shale gas

 d) Providing salt licks for wild animals in forests/protected areas

Show Answer

Answer: a 

Explanation:

• The ground-breaking technique known as "biorock technology" was developed in 1976 to create natural building materials from the sea. Coral reefs and other marine ecosystems, such as seagrass, salt marsh, mangroves, and oyster reefs, can survive and rebuild thanks to this special technique. 

• from physical devastation, climatic change, and damage brought on by an abundance of nutrients. All marine animals benefit substantially from it in terms of settlement, growth, survival, and resilience to environmental stressors like pollution and high temperatures.

Option (a) is the right response as a result. 

Q50. The “Miyawaki method” is well known for the:

 a) Promotion of commercial farming in arid and semi-arid areas

 b) Development of gardens using genetically modified flora

c) Creation of mini forests in urban areas

d) Harvesting wind energy on coastal areas and on sea surfaces

Show Answer

Answer c

Explanation:

• The Miyawaki forestation method is a novel approach to establishing an urban forest. It was developed by Japanese botanist Akira Miyawaki, who started planting young native plant species in tight groves to recover degraded areas in the 1970s. These species include trees, shrubs, and grasses. Tree Saplings

• When planted closely together, they compete for light and develop quickly. Miyawaki was engineering (and expediting) the stages of biological succession by planting a variety of native trees, mimicking the vegetation layers present in an established forest. This process naturally transforms a degraded plot into a forest.

Option (c) is the right response as a result.

Q51. In the Government of India Act 1919, the functions of Provincial Government were divided into "Reserved" and "Transferred" subjects.

 Which of the following were treated as "Reserved" subjects?

 1. Administration of Justice

2. Local Self-Government

3. Land Revenue

4. Police

Select the correct answer using the code given below:

 a) 1, 2 and 3

b) 2, 3 and 4

c) 1, 3 and 4

d) 1, 2 and 4

Show Answer

Answer: c

 Explanation:

• The Government of India Act 1919 established diarchy at the provincial level. It said that ministers were in charge of "transferred subjects," and the governor-in-council was in charge of "reserved subjects." Justice, land revenue, and finance were among the reserved topics.

• police, jail, relief from famine, irrigation, media, and public service. Local self-government, education, public health, agriculture, excise, public works, and cooperative organizations were among the subjects that were transferred.

Option (c) is the right response as a result.

Q52. In medieval India, the term "Fanam" referred to:

 a) Clothing

 b) Coins

 c) Ornaments

d) Weapons

Show Answer

Answer: b

 Explanation:

Fanam was a gold coin that was exclusively used in 17th and 18th century Southern India. The word Fanam is a Europeanized form of the word Panam, which meaning coin. Fanam was one of the tiniest coins ever made, and it was mostly made of gold. 0.35–0.4 grams (full) or half, between 0.15 and 0.2 grams. Trading was a major application for it.

• The Mughal Empire, Maratha, Mysore, Cochin, Coorg or Kurg, Travancore, Nagappal (for distribution in Ceylon), and Tuticorin are among the states who issued gold fanams. Up to 1815, the Madras Presidency also issued fanams. Fano was used in Danish India, whilst fanon was used in French India.

Option (b) is the right response as a result.

Q53. Consider the following freedom fighters:

 1. Barindra Kumar Ghosh

2. Jogesh Chandra Chatterjee

3. Rash Behari Bose

Who of the above was/were actively associated with the Ghadar Party?

 a) l and 2

b) 2 only

c) 1 and 3

d) 3 only

Show Answer

Answer: d

 Explanation:

• Barindra Kumar Ghosh did not take part in Ghadar since he was imprisoned during the First World War and after the Swadeshi years.

• After joining Anushilan Samiti, Jogesh Chandra Chatterjee went on to become one of the

original participants in HRA/HSRA. During the Ghadar era, only Rash Behari Bose was active. Rash Behari Bose, a local revolutionary leader, helped the Ghadar members when they first arrived in India with their attempt to incite an army uprising, which finally failed.

As a result, choice (d) is the right response.

Q54. With reference to the proposals of Cripps Mission, consider the following statements:

1. The Constituent Assembly would have members nominated by the Provincial Assemblies as well as the Princely States.

2. Any Province, which is not prepared to accept the new Constitution would have the right to sign a separate agreement with Britain regarding its future status.

Which of the statements given above is/are correct?

a) 1 only

b) 2 only

c) Both 1 and 2

 d) Neither 1 nor 2

Show Answer

Answer: b

Explanation:

In order to secure Indian support for the war, a delegation led by Stafford Cripps was dispatched to India in March 1942 with draft constitutional amendments.

• It was suggested that the Constituent Assembly should include of elected members, rather than province-nominated) members. The Princely states' portion was the only one that could receive nominations. Therefore, assertion 1 is untrue.

• It further declared that any Province that is unwilling to abide by the new Constitution may enter into a separate agreement with Britain as to its future status. This turned out to be the main cause of the Cripps mission's failure since it permitted India to become more divided. Therefore, statement 2 is true.

Option (b) is the right response as a result.

Q55. With reference to Indian history, consider the following texts:

 1. Nettipakarana

2. Parishishtaparvan

3. Avadanashataka

4. Trishashtilakshana Mahapurana

 Which of the above are Jaina texts?

 a) 1, 2 and 3

b) 2 and 4 only

c) 1, 3 and 4

d) 2, 3 and 4

Show Answer

Q56. With reference to Indian history, consider the following pairs:

Sl. No. Historical person Known as

1. Aryadeva : Jaina scholar

2. Dignaga : Buddhist scholar

3. Nathamuni : Vaishnava scholar

How many pairs given above are correctly matched?

 a) None of the pairs

b) Only one pair

c) Only two pairs

 d) All three pairs

Q56. With reference to Indian history, consider the following pairs:

Sl. No. Historical person Known as

1. Aryadeva : Jaina scholar

2. Dignaga : Buddhist scholar

3. Nathamuni : Vaishnava scholar

How many pairs given above are correctly matched?

 a) None of the pairs

b) Only one pair

c) Only two pairs

 d) All three pairs

Show Answer

Answer: b

Explanation:

• The Hokera Wetland is situated in Jammu and Kashmir's Union Territory. It is situated behind the snow-covered Pir Panchal in the northwest Himalayan biogeopgraphic province of Kashmir. Therefore, pair 1 is not matched correctly.

• Himachal Pradesh is home to Renuka Wetand. It is a naturally occurring marsh that is nourished by a little stream that runs from the lower Himalayan to the Giri river. It features freshwater springs and inland subterranean karst structures. Pair 2 is thus appropriately matched.

• The location of Rudrasagar Lake is in Tripura. Located in the northeastern highlands, this lowland sedimentation reservoir is supplied by three year-round streams that empty into the River Gomti. Pair 3 is thus appropriately matched.

• Kerala is where Sasthamkotta Lake is situated. It is the largest freshwater lake in Kerala state, which is located in the southwest of the nation. It is supplied by springs and provides drinking water to 500,000 people in the Kollam district. Pair 4 is therefore not well matched.

As a result, the right response is option (b).

Q57. With reference to Indian history, consider the following statements:

 1. The first Mongol Invasion of India happened during the reign of Jalal-ud-din Khalji.

 2. During the reign of Ala-ud-din Khalji, one Mongol assault marched up to Delhi and besieged the city. 3. Muhammad-bin-Tughlaq temporarily lost portions of north-west of his kingdom to Mongols.

 Which of the statements given above is/are correct?

a) 1 and 2

b) 2 only

c) 1 and 3

d) 3 only

Show Answer

Answer: b

Explanation:

• The Hokera Wetland is situated in Jammu and Kashmir's Union Territory. It is situated behind the snow-covered Pir Panchal in the northwest Himalayan biogeopgraphic province of Kashmir. Therefore, pair 1 is not matched correctly.

• Himachal Pradesh is home to Renuka Wetand. It is a naturally occurring marsh that is nourished by a little stream that runs from the lower Himalayan to the Giri river. It features freshwater springs and inland subterranean karst structures. Pair 2 is thus appropriately matched.

• The location of Rudrasagar Lake is in Tripura. Located in the northeastern highlands, this lowland sedimentation reservoir is supplied by three year-round streams that empty into the River Gomti. Pair 3 is thus appropriately matched.

• Kerala is where Sasthamkotta Lake is situated. It is the largest freshwater lake in Kerala state, which is located in the southwest of the nation. It is supplied by springs and provides drinking water to 500,000 people in the Kollam district. Pair 4 is therefore not well matched.

As a result, the right response is option (b).

Q58. With reference to Indian history, who of the following were known as "Kulah-Daran"?

a) Arab merchants

 b) Qalandars

c) Persian calligraphists

d) Sayyids

Show Answer

Answer: d

 Explanation

The Sayyids were known as Kulah-Daran (cap-wearers), and they belonged to the theological class during the Delhi Sultanate era. The characteristic pointed headgear that set them apart from the majority of theologians—who were officially known as Dastarbandan, or turban-wearers—was what made them stand out.

As a result, choice (d) is the right response.

Q59. With reference to Indian history, consider the following statements:

 1. The Dutch established their factories/warehouses on the east coast on lands granted to them by Gajapati rulers.

 2. Alfonso de Albuquerque captured Goa from the Bijapur Sultanate.

 3. The English East India Company established a factory at Madras on a plot of land leased from a representative of the Vijayanagara empire.

 Which of the statements given above are correct?

a) 1 and 2 only

 b) 2 and 3 only

c) 1 and 3 only

d) 1, 2 and 3

 

Show Answer

Explanation

• The Gajapati dynasty ruled over Odisha from 1434 till 1541. The following Dutch factories were founded on the eastern coast: Chinsura (1653), Bimilipatam/Bhimunipatnam (1641), Masulipattanam (1605), and Pulicat (1610). Therefore, assertion 1 is untrue.

• In 1510, Afonso de Albuquerque took Goa for the Portuguese from the Adil Shahi dynasty, who ruled Bijapur. Therefore, statement 2 is true.

• Local Raja Darmala Venkatadri Nayaka gave the English East India Company (EIC) permission to build a factory in Madras in 1639. In 1640, Fort Saint George was constructed. Madras was the first land the EIC leased, and notably, it was done amicably and via diplomacy rather than by force. The Vijayanagara Empire was represented by Raja Darmala Venkatadri. Thus, assertion 3 is true.

Option (b) is the right response as a result.

Q60. According to Kautilya’s Arthashastra, which of the following are correct?

1. A person could be a slave as a result of a judicial punishment.

2. If a female slave bore her master a son, she was legally free.

 3. If a son born to a female slave was fathered by her master, the son was entitled to the legal status of the master's son.

Which of the statements given above are correct?

 a) 1 and 2 only

b) 2 and 3 only

c) 1 and 3 only

d) 1, 2 and 3

Show Answer

Answer: b

Explanation:

• The Hokera Wetland is situated in Jammu and Kashmir's Union Territory. It is situated behind the snow-covered Pir Panchal in the northwest Himalayan biogeopgraphic province of Kashmir. Therefore, pair 1 is not matched correctly.

• Himachal Pradesh is home to Renuka Wetand. It is a naturally occurring marsh that is nourished by a little stream that runs from the lower Himalayan to the Giri river. It features freshwater springs and inland subterranean karst structures. Pair 2 is thus appropriately matched.

• The location of Rudrasagar Lake is in Tripura. Located in the northeastern highlands, this lowland sedimentation reservoir is supplied by three year-round streams that empty into the River Gomti. Pair 3 is thus appropriately matched.

• Kerala is where Sasthamkotta Lake is situated. It is the largest freshwater lake in Kerala state, which is located in the southwest of the nation. It is supplied by springs and provides drinking water to 500,000 people in the Kollam district. Pair 4 is therefore not well matched.

As a result, the right response is option (b).

Q61. Consider the following statements:

1. Tight monetary policy of US Federal Reserve could lead to capital flight.

2. Capital flight may increase the interest cost of firms with existing External Commercial Borrowings (ECBs).

 3. Devaluation of domestic currency decreases the currency risk associated with ECBs.

Which of the statements given above are correct?

 a) 1 and 2 only

 b) 2 and 3 only

c) 1 and 3 only

d) 1, 2 and 3

Show Answer

Answer: a

 Explanation:

A central bank may implement tight monetary policy to curb rapid economic expansion. Its objectives are to restrain expenditure in an economy that appears to be expanding too quickly or to slow down inflation when it appears to be increasing too quickly. When the United States Federal India may witness a flight of foreign money if the Reserve implements a strict monetary policy. This is a result of the US offering higher interest rates, which may draw in investors. Thus, the first statement is true.

• Capital flight weakens monetary management and destabilizes exchange and interest rates. It raises foreign borrowing's marginal costs. Thus, companies that already have external commercial borrowings may see a rise in interest costs as a result of capital flight. Thus, statement 2 is true.

• Currency devaluations raise the cost of borrowing for businesses that have debt denominated in foreign currencies, which harms their investments and net worth.

• Devaluation of the home currency, thus, raises the currency risk related to External

commercial loans. Therefore, assertion 3 is untrue.

Option (a) is the right response as a result.

Q62. Consider the following States:

1. Andhra Pradesh

2. Kerala

3. Himachal Pradesh

4. Tripura

How many of the above are generally known as tea-producing States?

a) Only one State

 b) Only two States

 c) Only three States

d) All four States

Show Answer

Answer: c

 Explanation:

• Most people associate the states of Kerala, Himachal Pradesh, and Tripura with tea production.

• The Anamallais are located just across the hills from the High Ranges, sandwiched between Tamil Nadu and Kerala. With more than 12,000 hectares planted to tea, it holds a significant position in the South Indian planting map. The Wayanad-Nilgiris range in Kerala, Tamil Nadu, and the Munnar region of Kerala are renowned for producing high-quality tea. Point 2 is thus accurate.

• Over 2,063 hectares, of tea is grown in the districts of Mandi and Kangra in Himachal Pradesh. Known as "the valley of gods," Kangra is well-known for its unique flavor of tea. Tea has been grown beneath the majestic and snow-capped Dhauladhar Mountain.

since 1949 on the mild inclines of the outer Himalayas. Point 3 is thus accurate.

• Tripura along with Assam is also one of the main tea growers in India. Thus, point 4 is accurate.

• Andhra Pradesh is primarily recognized for its coffee production, rather than being a state that produces tea. Therefore, point 1 is untrue.

Option (c) is the right response as a result.

Q63. Consider the following statements:

1. In India, credit rating agencies are regulated by Reserve Bank of India.

 2. The rating agency popularly known as ICRA is a public limited company.

 3. Brickwork Ratings is an Indian credit rating agency.

Which of the statements given above are correct?

 a) 1 and 2 only

 b) 2 and 3 only

c) 1 and 3 only

d) 1, 2 and 3

Show Answer

Option b

Explanation:

An organization that evaluates an issuer company's capacity and willingness to make on-time principal and interest payments on a debt instrument is called a credit rating agency. The Securities and Exchange Board of India oversees credit rating agencies (SEBI). Thus, claim 1 is untrue.

• As an independent and reputable investment information and credit rating agency, prominent financial and investment institutions, commercial banks, and financial services firms established ICRA Limited (previously Investment Information and Credit Rating Agency of India Limited) in 1991. ICRA is a public limited company, and the Bombay Stock Exchange and the National Stock Exchange both list its shares. Thus, statement 2 is true.

• credit rating company Brickwork Ratings (BWR) provides ratings on a variety of financial products and services, including bank loans, NCDs, commercial paper, fixed deposits, securitized paper, and security receipts.

Brickwork Ratings was established by former regulators, bankers, credit rating experts, and academics, and is dedicated to advancing financial literacy. The corporate office of BWR is located in Bengaluru. Therefore, statement 3 is true.

Option (b) is the right response as a result.

Q64. With reference to the ‘Banks Board Bureau (BBB), which of the following statements are correct? 1. The Governor of RBI is the Chairman of BBB.

 2. BBB recommends for the selection of heads for Public Sector Banks.

 3. BBB helps the Public Sector Banks in developing strategies and capital raising plans.

 Select the correct answer using the code given below:

 a) 1 and 2 only

 b) 2 and 3 only

c) 1 and 3 only

d) 1, 2 and 3

Show Answer

Answer: b

 Explanation:

• The Bank Board Bureau (BBB) Chairman is not the same as the Governor of the RBI. The Chairman and BBB members are chosen at the discretion of the Cabinet's Appointments Committee. Thus, the first statement is untrue.

• With regards to appointments, confirmation, tenure extensions, and termination of services of the Board of Directors of Public Sector Banks (PSBs) and financial institutions (FIs), the BBB provides advice to the Government. It is in charge of choosing and appointing the PSBs and FIs' boards of directors. Therefore, statement 2 is true.

• Assistance in the development of business strategies and capital raising plans, among other things, is one of the BBB's responsibilities for Public Sector Banks. Therefore, statement 3 is true.

Consequently, choice (b) is the right response as a result.

Q65. With reference to Convertible Bonds, consider the following statements:

1. As there is an option to exchange the bond for equity, Convertible Bonds pay a lower rate of interest. 2. The option to convert to equity affords the bondholder a degree of indexation to rising consumer prices.

Which of the statements given above is/are correct?

 a) 1 only

 b) 2 only

c) Both 1 and 2

d) Neither 1 nor 2

Show Answer

Answer: c

Explanation:

• The holder of a convertible bond has the choice to convert the bond or exchange it for a set number of shares in the issuing business. They function just like standard corporate bonds when they are issued, except with a little lower interest rate. Because convertibles might become stock companies offer lower rates on convertibles so that they can profit from an increase in the price of the underlying stock. Therefore, statement 1 is true.

• Convertible bonds have the benefit of giving bondholders some degree of indexation to growing consumer prices through the ability to convert to equity. Therefore, statement 2 is true.

Option (c) is therefore the right response.

Q66. Consider the following:

 1. Asian Infrastructure Investment Bank

 2. Missile Technology Control Regime

 3. Shanghai Cooperation Organisation

India is a member of which of the above?

a) 1 and 2 only

b) 3 only

c) 2 and 3 only

d) 1, 2 and 3

Show Answer

Answer: d

Explanation:

The goal of the multinational Asian Infrastructure Investment Bank (AIIB) is to finance infrastructure with a focus on sustainability, or building for tomorrow. Since launching in Beijing in January 2016, it has expanded to 105 authorized members across the globe. India joined the AIIB as a founding member. Thus, point 1 is accurate.

• India became an official member of the Missile Technology Control Regime (MTCR), a weapons regulatory body, in 2018. This allowed India access to sensitive technology and allowed it to export missiles around the world.

• The MTCR's objective is to limit the spread of missiles, full rocket systems, unmanned aerial vehicles, and associated technologies for those systems that can transport 500 kg of payloads across a minimum of 300 km, as well as systems meant for the transfer of WMDs, or weapons of mass devastation. Point 2 is thus accurate.

• On June 15, 2001, the Shanghai Cooperation Organization (SCO) was established as an intergovernmental organization in Shanghai. At the moment, there are eight member states of the SCO: China, India, Tajikistan, Kyrgyzstan, Russia, Pakistan, and Uzbekistan. In 2017, India was admitted to the SCO as a permanent member. Point 3 is thus accurate.

As a result, choice (d) is the right response.

Q67. Consider the following statements:

1. Vietnam has been one of the fastest growing economies in the world in the recent years.

 2. Vietnam is led by a multi-party-political system.

 3. Vietnam’s economic growth is linked to its integration with global supply chains and focus on exports. 4. For a long time, Vietnam’s low labour costs and stable exchange rates have attracted global manufacturers.

5. Vietnam has the most productive e-service sector in the Indo-Pacific region.

Which of the statements given above are correct?

 a) 2 and 4

b) 3 and 5

c) 1, 3 and 4

 d) 1 and 2

Show Answer

Answer: c

Explanation:

• By 2050, Vietnam's economy—which is among the fastest-growing in Southeast Asia—is predicted to rank among the top 20 in the world. Vietnam's GDP per capita climbed 3.6 times between 2002 and 2021, hitting about US$3,700. Only a few nations, including Vietnam, had GDP growth. when the pandemic struck in 2020. Therefore, statement 1 is true.

• Vietnam does not have a multiparty system. The Communist Party of Vietnam (CPV) is in charge of the socialist republic of Vietnam, an authoritarian state. Even though the Communist Party's ideological orthodoxy has lost some of its significance, the CPV still has a significant impact on the nation. There is insufficient authority separation between the legislative, executive, and judicial branches. There is a lot of overlap between the government and the CPV. Thus, statement 2 is untrue.

• Trade and investment have been key drivers of Vietnam’s impressive growth.

• Vietnam's ability to procure labor at a lower cost has allowed it to become a significant industrial hub in international supply chains. Vietnam's exports have also increased significantly. In April 2022, Vietnam's overall exports increased by 25.2% over the previous year. Vietnam's export industry, which is mostly supported by foreign direct investment, has benefited greatly from the steady currency rate that the central bank maintains within a 3 percent trading zone. As a result, claims 3 and 4 are true.

• In the Indo-Pacific area, Vietnam's e-service industry is not the most productive.

The e-service industries of nations like the United States, South Korea, and Japan are extremely developed and fruitful. Therefore, assertion 5 is untrue.

Option (c) is the right response as a result.

Q68. In India, which one of the following is responsible for maintaining price stability by controlling inflation?

 a) Department of Consumer Affairs

b) Expenditure Management Commission

c) Financial Stability and Development Council

 d) Reserve Bank of India

Show Answer

Answer: d

 Explanation:

The task of implementing monetary policy in India falls on the Reserve Bank of India (RBI). The Reserve Bank of India Act, 1934, expressly mandates this obligation. Mainly, monetary policy aims to preserve price stability while bearing in mind keep in mind the goal of development. Stable prices are an essential prerequisite for long-term, steady growth.

• The Reserve Bank of India (RBI) Act, 1934 was modified in May 2016 to give the flexible inflation targeting framework a legal foundation. The RBI Act has been revised to stipulate that the Government of India, in collaboration with the Reserve Bank, must determine the inflation target every five years.

As a result, choice (d) is the right response.

Q69. With reference to Non-Fungible Tokens (NFTs), consider the following statements:

1. They enable the digital representation of physical assets.

 2. They are unique cryptographic tokens that exist on a blockchain.

3. They can be traded or exchanged at equivalency and therefore can be used as a medium of commercial transactions.

 Which of the statements given above are correct?

a) 1 and 2 only

 b) 2 and 3 only

c) 1 and 3 only

d) 1, 2 and 3

Show Answer

Answer: a

 Explanation:

Non-fungible tokens, or NFTs, allow tangible assets to be represented digitally. A non-fungible token (NFT) is a digital asset that mimics tangible assets such as artwork, music, films, and in-game items. Therefore, statement 1 is true.

• They are typically encoded with the same underlying software as various cryptocurrencies and are bought and sold online, often with bitcoin. Typically, they are constructed using the same type of programming used to create cryptocurrencies like Bitcoin. NFTs are recorded transactions on a distributed public ledger called a blockchain. Therefore, statement 2 is true.

• Both fiat money and cryptocurrency are "fungible," which allows for trading or exchanging of one for the other. Additionally, their values are identical: one Bitcoin is always equal to another Bitcoin, and one dollar is always worth another dollar. Because cryptocurrency is fungible, it is a reliable way to carry out blockchain transactional operations. NFTs are not the same. Because each contains a digital signature, NFTs cannot be equalized or traded for one another; this renders them nonfungible. Therefore, assertion 3 is untrue.

Option (a) is the right response as a result.

Q29. In the northern hemisphere, the longest day of the year normally occurs in the:

 a) First half of the month of June

b) Second half of the month of June

c) First half of the month of July

d) Second half of the month of July

Show Answer

Answer: b

Explanation:

• The Hokera Wetland is situated in Jammu and Kashmir's Union Territory. It is situated behind the snow-covered Pir Panchal in the northwest Himalayan biogeopgraphic province of Kashmir. Therefore, pair 1 is not matched correctly.

• Himachal Pradesh is home to Renuka Wetand. It is a naturally occurring marsh that is nourished by a little stream that runs from the lower Himalayan to the Giri river. It features freshwater springs and inland subterranean karst structures. Pair 2 is thus appropriately matched.

• The location of Rudrasagar Lake is in Tripura. Located in the northeastern highlands, this lowland sedimentation reservoir is supplied by three year-round streams that empty into the River Gomti. Pair 3 is thus appropriately matched.

• Kerala is where Sasthamkotta Lake is situated. It is the largest freshwater lake in Kerala state, which is located in the southwest of the nation. It is supplied by springs and provides drinking water to 500,000 people in the Kollam district. Pair 4 is therefore not well matched.

As a result, the right response is option (b).

Sl. No. Reservoirs States
1 Ghataprabha : Telangana
2 Gandhi Sagar : Madhya Pradesh
3 Indira Sagar : Andhra Pradesh
4 Maithon : Chhattisgarh

How many pairs given above are not correctly matched?

 a) Only one pair

b) Only two pairs

 c) Only three pairs

 d) All four pairs

Show Answer

Answer: c

Explanation:

In the Gokak taluk of the Belgaum district of Karnataka, next to Ghataprabha hamlet, is the Ghataprabha reservoir, which is situated across the Ghataprabha river. The region's needs for drinking water and irrigation are met by the Ghataprabha reservoir. Therefore, pair 1 is not matched correctly.

• Madhya Pradesh is home to the Gandhi Sagar reservoir. It is among the five significant water reservoirs in the country. Gandhi Sagar Dam was built in 1960 to produce 115 megawatts of electricity and supply potable water to many districts in Rajasthan. Pair 2 is thus appropriately matched.

• The multifunctional Indira Sagar Dam is located in Madhya Pradesh, Central India. The Narmada River, which flows from Central to Western India, is where it was constructed. This multifunctional River Valley Project aims to develop the Narmada water resources. Pair 3 is therefore incorrectly matched.

• Maithon is situated alongside the Barakar River. About 48 kilometers separate the Maithon Dam in Jharkhand's coal city of Dhanbad. There isn't another dam like it in all of South East Asia, complete with an underground power station. Pair 4 is therefore not well matched.

Option (c) is the right response as a result.

Q71. In India, which one of the following compiles information on industrial disputes, closures, entrenchments and lay-offs in factories employing workers?

a) Central Statistics Office

b) Department for Promotion of Industry and Internal Trade

c) Labour Bureau

d) National Technical Manpower Information System

Show Answer

Answer: c

Explanation:

Since its founding in 1920, the Labour Bureau has been involved in the gathering, aggregation, and analysis as well as the distribution of labor data on various aspects of labor at the state and national levels throughout India.

• Voluntary submissions from the State Labor Departments and the Central Regional Labour Commissioner are used to construct the various versions of the statistics on "Industrial Disputes, Closures, Retrenchments and Layoffs in India during 2006."

Option (c) is the right response as a result.

Q72. In India, what is the role of the Coal Controller’s Organization (CCO)?

 1. CCO is the major source of Coal Statistics in Government of India.

 2. It monitors progress of development of Captive Coal/Lignite blocks.

 3. It hears any objection to the Government’s notification relating to acquisition of coal-bearing areas. 4. It ensures that coal mining companies deliver the coal to end users in the prescribed time.

 Select the correct answer using the code given below:

a) 1, 2 and 3

b) 3 and 4 only

c) 1 and 2 only

d) 1, 2 and 4

Show Answer

Answer: a

 Explanation:

The Coal Controller's Organization (CCO) has been designated as the statistical authority for coal and lignite statistics under the Collection of Statistics Act, 2008. the charge of conducting the annual coal and lignite survey, publishing the provisional coal statistics, and India Coal Directory. Additionally, it tracks the advancement of the captive coal/lignite blocks' development. Therefore, both claims 1 and 2 are true.

• The Central Government's notification on the purchase of coal-bearing land is subject to objections, and the CCO is the competent body under the Coal Bearing Area (purchase and Development) Act, 1957, to hear these objections and provide reports to the Central Government. Thus, statement 3 is accurate.

• It is not the responsibility of the CCO to make sure coal mining companies provide coal to end customers within the allotted time. Thus, statement 4 is untrue.

Option (a) is the right response as a result.

Q73. If a particular area is brought under the Fifth Schedule of the Constitution of India, which one of the following statements best reflects the consequence of it?

 a) This would prevent the transfer of land of tribal people to non-tribal people.

 b) This would create a local self-governing body in that area.

 c) This would convert that area into a Union Territory.

d) The State having such areas would be declared a Special Category State.

Show Answer

Answer: a

Explanation:

• The Constitution's Fifth Schedule addresses the management and governance of Scheduled Areas and Scheduled Tribes living in any state other than Assam, Meghalaya, Tripura, and Mizoram.

• Should a specific region be included in the Fifth Schedule, it implies that all land within the Scheduled Area is assumed to have been Adivasi land. Therefore, any land currently held by non-Adivasis should be returned to the Scheduled Tribes if it is being transferred.

Option (a) is the right response as a result.

Q74. Consider the following statements:

1. The India Sanitation Coalition is a platform to promote sustainable sanitation and is funded by the Government of India and the World Health Organization.

 2. The National Institute of Urban Affairs is an apex body of the Ministry of Housing and Urban Affairs in Government of India and provides innovative solutions to address the challenges of Urban India.

 Which of the statements given above is/are correct?

a) 1 only

 b) 2 only

c) Both 1 and 2

 d) Neither 1 nor 2

Show Answer

Answer: b

Explanation:

• The India Sanitation Coalition (ISC), established in June 2015 at the Federation of Indian Chambers of Commerce and Industry (FICCI), works to promote and facilitate sustainable and safe sanitation by uniting various groups through a variety of catalytic initiatives.

• These include organizing, curating, and sharing best practices in the sanitation advocacy space; supporting the unlocking of WASH financing with an emphasis on the private sector; forming alliances with organizations to spearhead the conversation on sustainable sanitation; and contributing to the policy aspects of sanitation by attending affiliated forums. Therefore, assertion 1 is untrue.

• The National Institute of Urban Affairs (NIUA), a flagship institute of the Ministry of Housing and Urban Affairs, Government of India, was founded in 1976 to advance research and develop the nation's urban sector's capabilities. It offers creative answers to the problems faced by urban India. Therefore, statement 2 is true.

Option (b) is the right response as a result.

Q75. Which one of the following has been constituted under the Environment (Protection) Act, 1986?

a) Central Water Commission

 b) Central Ground Water Board

 c) Central Ground Water Authority

d) National Water Development Agency

Show Answer

Answer: c

Explanation:

• The Environment (Protection) Act, 1986, Section 3(3), established the Central Ground Water Authority, whose purpose is to oversee and manage the nation's groundwater resources. It accomplishes this by issuing the relevant regulatory guidelines. It works out the authority granted by the Act to designate officers.

Option (c) is the right response as a result.

Q76. With reference to the “United Nations Credentials Committee”, consider the following statements:

1. It is a committee set up by the UN Security Council and works under its supervision.

2. It traditionally meets in March, June and September every year.

3. It assesses the credentials of all UN members before submitting a report to the General Assembly for approval.

 Which of the statements given above is/are correct?

 a) 3 only

 b) 1 and 3

c) 2 and 3

d) 1 and 2

Show Answer

Answer: a

Explanation:

• Each regular session of the General Assembly begins with the appointment of a United Nations Credentials Committee. Nine people make up this body; they are chosen by the General Assembly at the President's suggestion.

• The UN General Assembly grants the Credentials Committee its authority. Thus, the first statement is untrue.

• At the start of every General Assembly regular session, a Credentials Committee is appointed. The committee convenes in November and presents the report for adoption to the General Assembly in December. Thus, statement number two is untrue.

• According to Rule 28 of the General Assembly's Rules of Procedure, the Committee is tasked with reviewing the credentials of representatives of Member States and reporting to the Assembly on its findings. Therefore, statement 3 is true.

Option (a) is the right response as a result.

Q77. Which one of the following statements best describes the ‘Polar Code’?

 a) It is the international code of safety for ships operating in polar waters.

 b) It is the agreement of the countries around the North Pole regarding the demarcation of their territories in the polar region.

c) It is a set of norms to be followed by the countries whose scientists undertake research studies in the North Pole and South Pole.

 d) It is a trade and security agreement of the member countries of the Arctic Council.

Show Answer

Answer: d

Explanation:

• Non-member nations, international organizations, and other entities may be granted Observer Status by the UN General Assembly. Intergovernmental organizations can apply for UN General Assembly observer status. Thus, statements 1 and 2 are true.

• Observer status would only be available to States and multilateral organizations whose activity addresses topics of interest to the Assembly. Permanent observers are allowed to attend General Assembly sessions and proceedings and have missions at UN Headquarters. Therefore, statement 3 is true.

As a result, choice (d) is the right response.

Q78. With reference to the United Nations General Assembly, consider the following statements:

1. The UN General Assembly can grant observer status to the non-member States.

 2. Inter-governmental organisations can seek observer status in the UN General Assembly.

3. Permanent Observers in the UN General Assembly can maintain missions at the UN headquarters. Which of the statements given above are correct?

 a) 1 and 2 only

b) 2 and 3 only

 c) 1 and 3 only

d) 1, 2 and 3

Show Answer

Answer: d

 Explanation:

• Non-member nations, international organizations, and other entities may be granted Observer Status by the UN General Assembly. Intergovernmental organizations can apply for UN General Assembly observer status. Thus, statements 1 and 2 are true.

• Observer status would only be available to States and multilateral organizations whose activity addresses topics of interest to the Assembly. Permanent observers are allowed to attend General Assembly sessions and proceedings and have missions at UN Headquarters. Therefore, statement 3 is true.

As a result, choice (d) is the right response.

Q79. With reference to the “Tea Board” in India, consider the following statements:

 1. The Tea Board is a statutory body.

2. It is a regulatory body attached to the Ministry of Agriculture and Farmers Welfare.

 3. The Tea Board’s Head Office is situated in Bengaluru.

4. The Board has overseas offices at Dubai and Moscow.

 Which of the statements given above are correct?

a) 1 and 3

b) 2 and 4

c) 3 and 4

d) 1 and 4

Show Answer

Answer: d

Explanation:

Established on April 1st, 1954, the Tea Board is a statutory body created under section 4 of the Tea Act, 1953. Therefore, statement 1 is true.

• The current Tea Board operates as a Central Government statutory entity under the

Commerce Ministry. Therefore, assertion 2 is untrue.

• The Tea Board's headquarters are located in Kolkata. The Tea Board currently operates two offices abroad, one each in Moscow and Dubai. Its duties include providing technical and financial support for the production, marketing, and cultivation of tea; export promotion; and supporting R&D initiatives aimed at increasing tea output and raising tea standards. As a result, assertion 4 is true and statement 3 is false.

Option (d) is therefore the right one.

Q80. Which one of the following best describes the term “greenwashing”?

a) Conveying a false impression that a company’s products are eco-friendly and environmentally sound b) Non-inclusion of ecological/environmental costs in the Annual Financial Statements of a country

c) Ignoring the disastrous ecological consequences while undertaking infrastructure development

d) Making mandatory provisions for environmental costs in a government project/programme

Show Answer

Answer: b

Explanation:

• The Hokera Wetland is situated in Jammu and Kashmir's Union Territory. It is situated behind the snow-covered Pir Panchal in the northwest Himalayan biogeopgraphic province of Kashmir. Therefore, pair 1 is not matched correctly.

• Himachal Pradesh is home to Renuka Wetand. It is a naturally occurring marsh that is nourished by a little stream that runs from the lower Himalayan to the Giri river. It features freshwater springs and inland subterranean karst structures. Pair 2 is thus appropriately matched.

• The location of Rudrasagar Lake is in Tripura. Located in the northeastern highlands, this lowland sedimentation reservoir is supplied by three year-round streams that empty into the River Gomti. Pair 3 is thus appropriately matched.

• Kerala is where Sasthamkotta Lake is situated. It is the largest freshwater lake in Kerala state, which is located in the southwest of the nation. It is supplied by springs and provides drinking water to 500,000 people in the Kollam district. Pair 4 is therefore not well matched.

As a result, the right response is option (b).

Q81. Consider the following statements:

1. High clouds primarily reflect solar radiation and cool the surface of the Earth.

2. Low clouds have a high absorption of infrared radiation emanating from the Earth’s surface and thus cause warming effect.

 Which of the statements given above is/are correct?

a) 1 only

b) 2 only

c) Both 1 and 2

d) Neither 1 nor 2

Show Answer

Answer: d

 Explanation:

Earth typically warms more while clouds are high in the atmosphere than when they cool. Slender, tall clouds retain a portion of the Sun's heat. Earth's surface warms as a result. Therefore, assertion 1 is untrue.

• Low clouds tend to chill more than warm when they are located within a mile or two of the Earth's surface. The heat from the Sun is largely reflected by lower, denser clouds. Thus, the surface of the Earth cools. Therefore, assertion 2 is untrue. 

As a result, choice (d) is the right response. ).

Q82. Consider the following statements:

1. Bidibidi is a large refugee settlement in north-western Kenya.

2. Some people who fled from South Sudan civil war live in Bidibidi.

3. Some people who fled from civil war in Somalia live in Dadaab refugee complex in Kenya.

Which of the statements given above is/are correct?

a) 1 and 2

b) 2 only

c) 2 and 3

d) 3 only

Show Answer

Answer: c

 Explanation:

• The second-largest refugee community in the world, the Bidibidi Refugee community is home to around 270,000 South Sudanese refugees and is situated in Uganda's West Nile region. As a result, assertion 2 is true and statement 1 is false.

• More than 200,000 refugees and asylum seekers are officially registered at the Dadaab refugee complex. There are three camps in the Dadaab refugee complex. When refugees began to cross the border into Kenya in 1991 to escape the civil violence in Somalia, the first camp was set up. Thus, assertion 3 is true.

Option (c) is the right response as a result.

Q83. Consider the following countries:

 1. Armenia

2. Azerbaijan

 3. Croatia

4. Romania

5. Uzbekistan

Which of the above are members of the Organization of Turkic States?

a) 1, 2 and 4

b) 1 and 3

c) 2 and 5

d) 3, 4 and 5

Show Answer

Answer: c

 Explanation:

• The main objective of the 2009-founded Organization of Turkic States, formerly known as the collaboration Council of Turkic Speaking States - Turkic Council, was to foster comprehensive collaboration among Turkic States. 

• Azerbaijan, Kazakhstan, Kyrgyzstan, and Turkey are its founding members. In October 2019, during the 7th Summit in Baku, Uzbekistan became a full member. In September 2018, during the Organization's Sixth Summit in Cholpon-Ata, Kyrgyz Republic, Hungary was granted observer status. Additionally, Turkmenistan has joined the organization as an observer member at the 8th Summit, which took place in November 2021. Thus, the points 2 and 5 are accurate. 

• Its members are not Romania, Croatia, or Armenia. Therefore, points 1, 3, and 4 are false. 

Option (c) is the right response as a result. • The location of Rudrasagar Lake is in Tripura. Located in the northeastern highlands, this lowland sedimentation reservoir is supplied by three year-round streams that empty into the River Gomti. Pair 3 is thus appropriately matched.

• Kerala is where Sasthamkotta Lake is situated. It is the largest freshwater lake in Kerala state, which is located in the southwest of the nation. It is supplied by springs and provides drinking water to 500,000 people in the Kollam district. Pair 4 is therefore not well matched.

As a result, the right response is option (b).

Q84. Consider the following statements:

1. Gujarat has the largest solar park in India.

 2. Kerala has a fully solar powered International Airport.

 3. Goa has the largest floating solar photovoltaic project in India.

 Which of the statements given above is/are correct?

 a) 1 and 2

 b) 2 only

c) 1 and 3

d) 3 only

Show Answer

Answer: b

 Explanation:

The biggest solar power park in the world is located in India, at the Bhadla Solar Park. The 14,000-acre Bhadla Solar Park is situated in the arid and sandy region of Rajasthan. Therefore, assertion 1 is untrue.

• Cochin International Airport, the first airport in India constructed using the public-private partnership (PPP) model, has written another chapter in the history of aviation by being the first airport globally to run entirely on solar energy. Therefore, statement 2 is true.

• The Simhadri reservoir in Andhra Pradesh is home to NTPC's floating solar project, which takes up more than 75 acres. It will use over a lakh solar PV modules to provide electricity to power 7,000 households. Therefore, assertion 3 is untrue.

Option (b) is the right response as a result.

Q85. With reference to the United Nations Convention on the Law of Sea, consider the following statements:

1. A coastal state has the right to establish the breadth of its territorial sea up to a limit not exceeding 12 nautical miles, measured from baseline determined in accordance with the convention.

 2. Ships of all states, whether coastal or land-locked, enjoy the right of innocent passage through the territorial sea.

 3. The Exclusive Economic Zone shall not extend beyond 200 nautical miles from the baseline from which the breadth of the territorial sea in measure.

Which of the statements given above are correct?

a) 1 and 2 only

b) 2 and 3 only

c) 1 and 3 only

d) 1, 2 and 3

Show Answer

Answer: d

Explanation:

• 1982 saw the adoption of the United Nations Convention on the Law of the Sea. It establishes regulations governing all uses of the oceans and their resources and lays out a complete system of law and order in the world's oceans and seas.

• It states that each State may select the length of its territorial sea as long as it is measured from baselines established in compliance with this Convention and does not exceed 12 nautical miles. Therefore, statement 1 is true.

• Ships of all States, coastal or landlocked, have the right to transit through their territorial seas in peace under the terms of this convention. Thus, statement 2 is true.

• It specifies that the exclusive economic zone cannot be extended farther than 200 nautical miles from the baselines used to gauge the territorial sea's width. Thus, statement 3 is true.

As a result, choice (d) is the right response.

Q86. Which one of the following statements best reflects the issue with Senkaku Islands, sometimes mentioned in the news?

 a) It is generally believed that they are artificial islands made by a country around South China Sea.

 b) China and Japan engage in maritime disputes over these islands in East China Sea.

 c) A permanent American military base has been set up there to help Taiwan to increase its defence capabilities.

d) Though International Court of Justice declared them as no man’s land, some South-East Asian countries claim them.

Show Answer

Answer: b

Explanation:

• After being legally claimed by Japan in 1895, several Japanese nationals have privately controlled the Senkaku/Diaoyu islands for the majority of the previous 120 years. Citing historical evidence, China reaffirmed its claims to the Senkaku/Diaoyu islands in the 1970s.

• As both nations enhance their military capabilities, notably their radar and missile systems, in the region, tensions between China and Japan over the disputed Senkaku/Diaoyu islands continue to rise.

Option (b) is the right response as a result.

Lorem ipsum dolor sit amet, consectetuer adipiscing elit, sed diam nonummy nibh euismod tincidunt ut laoreet dolore magna aliquam erat volutpat.

Sl. No. Country Important reason for being in the news recently
1 Chad : Setting up of permanent military base by China
2 Guinea : Suspension of Constitution and Government by military
3 Lebanon : Severe and prolonged economic depression
4 Tunisia : Suspension of Parliament by President

How many pairs given above are correctly matched?

a) Only one pair

b) Only two pairs

c) Only three pairs

d) All four pairs


Show Answer

Answer: b

Explanation:

Dhauli is situated in what was once Kalinga, the modern state of Odisha, which was subjugated by the monarch Ashoka Maurya (r. 272-231 BC) in roughly 260 BC with a terrible death toll. Following that, Ashoka expressed regret for his acts of violence and turned to Buddhism. In a series of edicts carved into rock, he confessed his guilt and declared his determination to rule the kingdom by the precepts of his new faith. Pair 1 is thus appropriately matched.

• The Ashoka Rock Edict site is located in the Kurnool district of Andhra Pradesh, close to Erragudi on the Gooty-Pathikonda route. Pair 2 is thus appropriately matched.

• The Kalinga province's Mauryan fortified capital was Jaugada, a historic fort.

• Jaugada is situated in the Ganjam district of Odisha, close to the cities of Purushottampur and Berhampur. This location is well-known in Odisha for Kalinga edicts in addition to Dhauli. Consequently, pair 3 is not accurately matched.

• King Asoka's rock edicts of Kalsi, situated roughly 43 kilometers from Dehradun in Uttarakhand. Pair 4 is therefore not well matched.

Option (b) is the right response as a result.

Q89. With reference to Indian laws about wildlife protection, consider the following statements:

 1. Wild animals are the sole property of the government.

2. When a wild animal is declared protected, such animal is entitled for equal protection whether it is found in protected areas or outside.

3. Apprehension of a protected wild animal becoming a danger to human life is sufficient ground for its capture or killing.

Which of the statements given above is/are correct?

a) 1 and 2

b) 2 only

c) 1 and 3

d) 3 only

Show Answer

Answer: a

 Explanation:

The Wildlife Protection Act of 1972 states that wild animals belong to the government (either the state or the central government). The Bombay High Court rendered a landmark decision in 2012, holding that tigers and other wild animals should be regarded as government property for all intents and purposes and the government need to make up for any harm they do. Thus, the first statement is true.

• There is no distinction made between animals found in protected areas and those found outside by the Wildlife (Protection) Act of 1972. It offers all wild creatures the same level of protection regardless of where they are located. Therefore, statement 2 is true.

• A protected wild animal cannot be captured or killed just because someone fears it poses a threat to human safety. The wild animal may only be taken into custody or put to death by the authorities if it poses a threat to human safety, is incurably sick or is otherwise incapacitated by competent authority, such as the State's Chief Wildlife Warden. Therefore, assertion 3 is untrue.

Option (a) is the right response as a result.

Q90. Certain species of which one of the following organisms are well known as cultivators of fungi?

 a) Ant

b) Cockroach

c) Crab

d) Spider

Show Answer

Answer: a

Explanation:

• A diverse range of agricultural practices are employed by the more than 200 species of attine ants to cultivate mutualistic fungi in fungus gardens. The fungi grown by leaf-cutters are incapable of growing freely, in contrast to those grown by lesser attines, and rely only on the ants to spread across the colony. Like the fruits and vegetables we grow, these fungi also develop nutrient-dense swellings that the ants eat. To put it simply, the ants have tamed them.

Option (a) is the right response as a result.

Q91. Consider the following pairs:

Sl. No. Site of Ashoka’s Major Rock Edicts Location in the State of
1 Dhauli : Odisha
2 Erragudi : Andhra Pradesh
3 Jaugada : Madhya Pradesh
4 Kalsi : Karnataka

How many pairs given above are correctly matched?

a) Only one pair

b) Only two pairs

c) Only three pairs

d) All four pairs

Show Answer

Answer: b

Explanation:

Dhauli is situated in what was once Kalinga, the modern state of Odisha, which was subjugated by the monarch Ashoka Maurya (r. 272-231 BC) in roughly 260 BC with a terrible death toll. Following that, Ashoka expressed regret for his acts of violence and turned to Buddhism. In a series of edicts carved into rock, he confessed his guilt and declared his determination to rule the kingdom by the precepts of his new faith. Pair 1 is thus appropriately matched.

• The Ashoka Rock Edict site is located in the Kurnool district of Andhra Pradesh, close to Erragudi on the Gooty-Pathikonda route. Pair 2 is thus appropriately matched.

• The Kalinga province's Mauryan fortified capital was Jaugada, a historic fort.

• Jaugada is situated in the Ganjam district of Odisha, close to the cities of Purushottampur and Berhampur. This location is well-known in Odisha for Kalinga edicts in addition to Dhauli. Consequently, pair 3 is not accurately matched.

• King Asoka's rock edicts of Kalsi, situated roughly 43 kilometers from Dehradun in Uttarakhand. Pair 4 is therefore not well matched.

Option (b) is the right response as a result.

Q91. Consider the following pairs:

Sl. No. King Dynasty
1 Nannuka : Chandela
2 Jayashakti : Paramara
3 Nagabhata II : Gurajara-Pratihara
4 Bhoja : Rashtrakuta

How many pairs given above are correctly matched?

a) Only one pair

b) Only two pairs

c) Only three pairs

d) All four pairs

Show Answer

Answer: b

Explanation:

Dhauli is situated in what was once Kalinga, the modern state of Odisha, which was subjugated by the monarch Ashoka Maurya (r. 272-231 BC) in roughly 260 BC with a terrible death toll. Following that, Ashoka expressed regret for his acts of violence and turned to Buddhism. In a series of edicts carved into rock, he confessed his guilt and declared his determination to rule the kingdom by the precepts of his new faith. Pair 1 is thus appropriately matched.

• The Ashoka Rock Edict site is located in the Kurnool district of Andhra Pradesh, close to Erragudi on the Gooty-Pathikonda route. Pair 2 is thus appropriately matched.

• The Kalinga province's Mauryan fortified capital was Jaugada, a historic fort.

• Jaugada is situated in the Ganjam district of Odisha, close to the cities of Purushottampur and Berhampur. This location is well-known in Odisha for Kalinga edicts in addition to Dhauli. Consequently, pair 3 is not accurately matched.

• King Asoka's rock edicts of Kalsi, situated roughly 43 kilometers from Dehradun in Uttarakhand. Pair 4 is therefore not well matched.

Option (b) is the right response as a result.

Q93. Which one of the following statements about Sangam literature in ancient South India is correct?

 a) Sangam poems are devoid of any reference to material culture.

 b) The social classification of Varna was known to Sangam poets.

 c) Sangam poems have no reference to warrior ethic.

 d) Sangam literature refers to magical forces as irrational.

Show Answer

Answer: b

 Explanation:

The collection of poetry known as Sangam literature was composed by Tamils from a wide range of social backgrounds over six centuries, from about 300 BC to 300 AD. It took several centuries for it to be collected into anthologies. It provides details on material culture in those times in addition to describing significant fighters. These writings shed light on the pre-Tamil culture and the commercial connections between South India and West Asia, Southeast Asia, and the Mediterranean. Sangam poets were aware of Varna's social stratification. We find a Sutra (74) in the Purattinai of Tolkappiyam that refers to four Varnas by the name Vahai (type).

Option (b) is the right response as a result.

Q94. “Yogavasistha” was translated into Persian by Nizamuddin Panipati during the reign of:

 a) Akbar

 b) Humayun

c) Shahjahan

d) Aurangzeb

Show Answer

Answer: a

Explanation:

• Yogavasistha, commonly referred to as the Maha-Ramayana, is a well-known and significant Hindu syncretic philosophical work from the past. During Akbar's reign, Nizamuddin Panipati, who served as his Mir Bakshi, translated it into Persian.

Option (a) is therefore the right response.

Q95. The world’s second tallest statue in sitting pose of Ramanuja was inaugurated by the Prime Minister of India at Hyderabad recently. Which one of the following statements correctly represents the teachings of Ramanuja?

 a) The best means of salvation was devotion.

b) Vedas are eternal, self-existent and wholly authoritative.

c) Logical arguments were essential means for the highest bliss.

 d) Salvation was to be obtained through meditation.

Show Answer

Answer: a

Explanation:

Born in Sriperumbudur, Tamil Nadu, around 1017, Saint Ramanujacharya was a philosopher and social reformer who traveled all across India promoting social justice and equality.

• He promoted equality, humility, compassion, and dedication to God as means of achieving universal salvation and regard for one another, which is referred to as Sri Vaishnavam Sampradaya. Ramanuja brought the Bhakti movement back to life, and subsequent Bhakti schools of thought were influenced by his teachings. He is credited with inspiring poets such as Bhakt Ramdas, Annamacharya, Thyagaraja, Kabir, and Meerabai.

Option (a) is the right response as a result.

Q96. The Prime Minister recently inaugurated the new Circuit House near Somnath Temple at Veraval. Which of the following statements are correct regarding Somnath Temple?

 1. Somnath Temple is one of the Jyotirlinga shrines.

 2. A description of Somnath Temple was given by Al-Biruni.

 3. Pran Pratishtha of Somnath Temple (installation of the present day temple) was done by President S. Radhakrishnan.

Select the correct answer using the code given below:

a) 1 and 2 only

b) 2 and 3 only

c) 1 and 3 only

d) 1, 2 and 3

Show Answer

Answer: a

 Explanation:

Out of the twelve Aadi Jyotirlings of India, Shree Somnath is ranked first. On India's western coast, it is in a strategically important position. It is thought that the Somnath temple is the first of Lord Shiva's twelve Jyotirlinga shrines. Therefore, statement 1 is true.

• The temple was reported by Al Biruni, an Arab traveler from the eleventh century, who also mentioned its destruction and looting. Therefore, statement 2 is true.

The Pran-Pratistha at the current temple was performed on May 11, 1951, by Dr. Rajendra Prasad, the Indian President at the time. Therefore, statement 3 is true.

Option (a) is the right response as a result.

Q97. Which one of the following statements best describes the role of B cells and T cells in the human body?

 a) They protect the body from environmental allergens.

 b) They alleviate the body's pain and inflammation.

 c) They act as immunosuppressants in the body.

d) They protect the body from the diseases caused by pathogens.

Show Answer

Answer: b

Explanation:

• The Hokera Wetland is situated in Jammu and Kashmir's Union Territory. It is situated behind the snow-covered Pir Panchal in the northwest Himalayan biogeopgraphic province of Kashmir. Therefore, pair 1 is not matched correctly.

• Himachal Pradesh is home to Renuka Wetand. It is a naturally occurring marsh that is nourished by a little stream that runs from the lower Himalayan to the Giri river. It features freshwater springs and inland subterranean karst structures. Pair 2 is thus appropriately matched.

• The location of Rudrasagar Lake is in Tripura. Located in the northeastern highlands, this lowland sedimentation reservoir is supplied by three year-round streams that empty into the River Gomti. Pair 3 is thus appropriately matched.

• Kerala is where Sasthamkotta Lake is situated. It is the largest freshwater lake in Kerala state, which is located in the southwest of the nation. It is supplied by springs and provides drinking water to 500,000 people in the Kollam district. Pair 4 is therefore not well matched.

As a result, the right response is option (b).

Q98. Consider the following statements:

 1. Other than those made by humans, nanoparticles do not exist in nature.

 2. Nanoparticles of some metallic oxides are used in manufacture of some cosmetics

3. Nanoparticles of some commercial products which enter environment are unsafe for humans.

 Which of the statements given above is/are correct?

 a) 1 only

 b) 3 only

c) 1 and 2

d) 2 and 3

Show Answer

Answer: d

 Explanation:

Extremely small fragments of a particular material are known as nanoparticles. Nanoparticles have a thickness of less than 100 nm. A nanometer is 1000 times smaller than the thickness of a single hair strand, to put things into perspective.

• Natural sources of nanoparticles include microscopic sea spray droplets.

However, the majority of nanoparticles are made in laboratories. The nanoparticles in question for sunscreen include titanium dioxide and zinc oxide. Before being incorporated into your sunscreen, these components are reduced to extremely small particles. Thus, while assertion 2 is true, statement 1 is false.

• Breathed particulate matter can accumulate throughout the respiratory system, with the lungs receiving a significant portion of inhaled nanoparticles. Nanoparticles can travel from the lungs to other organs such as the brain, liver, spleen, and possibly the fetus in expectant mothers. Therefore, statement 3 is true.

As a result, choice (d) is the right response.

Q99. Consider the following statements:

DNA Barcoding can be a tool to:

 1. assess the age of a plant or animal.

 2. distinguish among species that look alike.

 3. identify undesirable animal or plant materials in processed foods.

Which of the statements given above is/are correct?

a) 1 only

 b) 3 only

 c) 1 and 2

d) 2 and 3

Show Answer

Answer: d

Explanation:

• DNA barcoding is a method of identifying species that relies on the usage of a brief, standardized genomic region that functions as a "barcode," much like supermarket scanners use UPCs to identify commercial goods. There is no DNA barcoding correlation with establishing an organism's age. Therefore, assertion 1 is untrue.

• For eukaryotes, DNA barcoding is a method for quick and precise species identification that uses a short DNA sequence rather than the entire genome to increase accessibility to ecological systems. Therefore, statement 2 is true.

• The most effective method for detecting food adulteration with undesired plant and animal material in recent times is DNA barcoding. Therefore, statement 3 is true.

As a result, choice (d) is the right response.

Q100. Consider the following:

 1. Carbon monoxide

2. Nitrogen oxide

3. Ozone

4. Sulphur dioxide

Excess of which of the above in the environment is/are cause(s) of acid rain?

 a) 1, 2 and 3

b) 2 and 4 only

c) 4 only

d) 1, 3 and 4

Show Answer

Answer: b

 Explanation:

The term "acid rain" describes the processes by which acid from the atmosphere settles on the surface of the earth. Acidic oxides of sulfur and nitrogen can be carried by the wind and land on solid particles in the atmosphere or on the ground as dry matter deposition or as wet deposition in water, fog, and snow.

• Sulfur dioxide and nitrogen oxides are produced when fossil fuels, such as coal and oil in power plants and furnaces or gasoline and diesel in motor engines, burn. These fuels contain sulfur and nitrogenous materials. Because particulate matter in contaminated air typically catalyzes oxidation, SO2 and NO2 following oxidation and interaction with water are primary contributors to acid rain.

As a result Option (b) is the right answer.

Prelims

More Year Papers

Get in Touch

Bringing Your Vision To Life

View Details
- +
Sold Out